Сложные примеры логарифмы: Логарифмы примеры решения задач, формулы и онлайн калькуляторы

Содержание

Логарифмы примеры решения задач, формулы и онлайн калькуляторы

Содержание:

Логарифмы (Логарифмирование) активно используются в решении задач, так как значительно упрощают обычные алгебраические операции.
Использование логарифмов позволяет заменить умножение на значительно более простое сложение, деление — на вычитание,
а возведение в степень и извлечение корня заменяются соответственно на умножение и деление на показатель степени числа.

Перед изучением примеров решения задач советуем изучить теоретический материал по логарифмам, прочитать определения и
все свойства логарифмов.

Логарифм произведения, сумма логарифмов

Теоретический материал по теме — логарифм произведения.

Пример

Задание. Представить $\log _{5} 6$ в виде суммы логарифмов.

Решение. $\log _{5} 6=\log _{5}(2 \cdot 3)=\log _{5} 2+\log _{5} 3$

Слишком сложно?

Примеры решения задач с логарифмами не по зубам? Тебе ответит эксперт через 10 минут!

Пример

Задание. Упростить $\log _{5} 4+\log _{5} 3$

Решение. $\log _{5} 4+\log _{5} 3=\log _{5}(4 \cdot 3)=\log _{5} 12$

Логарифм частного, разность логарифмов

Теоретический материал по теме — логарифм частного.

Пример

Задание. Известно, что $\log _{5} 2=a$,
а $\log _{5} 3=b$. Выразить $\log _{5} \frac{2}{3}$ через $a$ и $b$.

Решение. $\log _{5} \frac{2}{3}=\log _{5} 2-\log _{5} 3=a-b$

Пример

Задание. Вычислить значение выражения $\log _{5} 10-\log _{5} 2$

Решение. $\log _{5} 10-\log _{5} 2=\log _{5} \frac{10}{2}=\log _{5} 5=1$

Логарифм степени

Теоретический материал по теме — логарифм степени.

Пример

Задание. Вычислить $\log _{5} 10-\log _{5} 2=\log _{5} \frac{10}{2}=\log _{5} 5=1$

Решение.$\log _{2} \frac{1}{8}+\log _{5} 25=\log _{2} 2^{-3}+\log _{5} 5^{2}=-3 \cdot \log _{2} 2+2 \cdot \log _{5} 5=$

$= -3 + 2 = -1$

Пример

Задание. {2}-x-2=0 \Rightarrow x_{1}=2, x_{2}=-1$$

Второй корень не принадлежит ОДЗ, а значит решение $x=2$

Ответ. $x=2$

Пример

Задание. Решить уравнение $\ln (x+1)=\ln (2 x-3)$

Решение. Находим ОДЗ:

$$\left\{\begin{array}{l}
x+1>0 \\
2 x-3>0
\end{array} \Rightarrow\left\{\begin{array}{l}
x>-1 \\
2 x>3
\end{array} \Rightarrow\left\{\begin{array}{l}
x>-1 \\
x>\frac{3}{2}
\end{array} \Rightarrow\left(\frac{3}{2} ;+\infty\right)\right.\right.\right.$$

Решаем уравнение $x+1=2 x-3: x=4 \in$ ОДЗ.

Итак, решением исходного логарифмического уравнения
также является это значение.

Ответ. $x=4$

Решение логарифмических неравенств

Теоретический материал по теме — логарифмические неравенства.

Пример

Задание. Решить неравенство $\log _{0,5}(x-1)>-1$

Решение. {-1}$   или   $x-1<2 \Rightarrow x<3$

В пересечении с ОДЗ получаем, что $x \in(1 ; 3)$

Ответ. $x \in(1 ; 3)$

Пример

Задание. Решить неравенство $\log _{5} 5>\log _{5} x$

Решение. Данное неравенство равносильно системе:

$$\left\{\begin{array}{l}
5>x, \\
x>0
\end{array} \Rightarrow\left\{\begin{array}{l}
x0
\end{array} \Rightarrow x \in(0 ; 5)\right.\right.$$

Ответ. $x \in(0 ; 5)$

Читать первую тему — формулы и свойства логарифмов,
раздела логарифмы.

Вычисление логарифмов: способы, примеры, решения

Вычисление логарифмов по определению

В простейших случаях возможно достаточно быстро и легко выполнить нахождение логарифма по определению. Давайте подробно рассмотрим, как происходит этот процесс.

Его суть состоит в представлении числа b в виде ac, откуда по определению логарифма число c является значением логарифма. То есть, нахождению логарифма по определению отвечает следующая цепочка равенств: logab=logaac=c.

Итак, вычисление логарифма по определению сводится к нахождению такого числа c, что ac=b, а само число c есть искомое значение логарифма.

Учитывая информацию предыдущих абзацев, когда число под знаком логарифма задано некоторой степенью основания логарифма, то можно сразу указать, чему равен логарифм – он равен показателю степени. Покажем решения примеров.

Найдите log22−3, а также вычислите натуральный логарифм числа e5,3.

Определение логарифма позволяет нам сразу сказать, что log22−3=−3. Действительно, число под знаком логарифма равно основанию 2 в −3 степени.

Аналогично находим второй логарифм: lne5,3=5,3.

log22−3=−3 и lne5,3=5,3.

Если же число b под знаком логарифма не задано как степень основания логарифма, то нужно внимательно посмотреть, нельзя ли прийти к представлению числа b в виде ac. Часто такое представление бывает достаточно очевидно, особенно когда число под знаком логарифма равно основанию в степени 1, или 2, или 3, …

Вычислите логарифмы log525, и .

log525=2, и .

Когда под знаком логарифма находится достаточно большое натуральное число, то его не помешает разложить на простые множители. Это часто помогает представить такое число в виде некоторой степени основания логарифма, а значит, вычислить этот логарифм по определению.

В заключение этого пункта отметим, что мы не ставили целью рассмотреть все способы представления числа под знаком логарифма в виде некоторой степени основания. Наша цель заключалась в том, чтобы дать самые часто используемые варианты действий, приводящие к результату при вычислении логарифмов по определению.

Как решать «вложенные» логарифмические уравнения

Сегодня мы продолжаем изучать логарифмические уравнения и разберем конструкции, когда один логарифм стоит под знаком другого логарифма. Оба уравнения мы будем решать с помощью канонической формы.Сегодня мы продолжаем изучать логарифмические уравнения и разберем конструкции, когда один логарифм стоит под знаком другого. Оба уравнения мы будем решать с помощью канонической формы. Напомню, если у нас есть простейшее логарифмическое уравнение вида logaf(x) = b, то для решения такого уравнения мы выполняем следующие шаги. В первую очередь, нам нужно заменить число b:

b = logaab

Заметьте: ab— это аргумент. Точно так же в исходном уравнении аргументом является функция f(x). Затем мы переписываем уравнение и получаем вот такую конструкцию:

logaf(x) = logaab

Уже затем мы можем выполнить третий шаг — избавится от знака логарифма и просто записать:

f(x) = ab

В результате мы получим новое уравнение. При этом никаких ограничений на функцию f(x) не накладывается. Например, на ее месте также может стоять логарифмическая функция. И тогда мы вновь получим логарифмическое уравнение, которое снова сведем к простейшему и решим через каноническую форму.

Впрочем, хватит лирики. Давайте решим настоящую задачу. Итак, задача № 1:

log2 (1 + 3 log2x) = 2

Как видим, перед нами простейшее логарифмическое уравнение. В роли f(x) выступает конструкция 1 + 3 log2x, а в роли числа b выступает число 2 (в роли aтакже выступает двойка). Давайте перепишем эту двойку следующим образом:

2 = log2 22

Важно понимать, что первые две двойки пришли к нам из основания логарифма, т. е. если бы в исходном уравнении стояла 5, то мы бы получили, что 2 = log5 52. В общем, основание зависит исключительно от логарифма, который изначально дан в задаче. И в нашем случае это число 2.

Итак, переписываем наше логарифмическое уравнение с учетом того, что двойка, которая стоит справа, на самом деле тоже является логарифмом. Получим:

log2 (1 + 3 log2x) = log2 4

Переходим к последнему шагу нашей схемы — избавляемся от канонической формы. Можно сказать, просто зачеркиваем знаки log. Однако с точки зрения математики «зачеркнуть log» невозможно — правильнее сказать, что мы просто просто приравниваем аргументы:

1 + 3 log2x = 4

Отсюда легко находится 3 log2x:

3 log2x = 3

log2x = 1

Мы вновь получили простейшее логарифмическое уравнение, давайте снова приведем его к канонической форме. Для этого нам необходимо провести следующие изменения:

1 = log2 21 = log2 2

Почему в основании именно двойка? Потому что в нашем каноническом уравнении слева стоит логарифм именно по основанию 2. Переписываем задачу с учетом этого факта:

log2x = log2 2

Снова избавляемся от знака логарифма, т. е. просто приравниваем аргументы. Мы вправе это сделать, потому что основания одинаковые, и больше никаких дополнительных действий ни справа, ни слева не выполнялось:

х = 2

Вот и все! Задача решена. Мы нашли решение логарифмического уравнения.

Обратите внимание! Хотя переменная х и стоит в аргументе (т. е. возникают требования к области определения), мы никаких дополнительных требований предъявлять не будем.

Как я уже говорил выше, данная проверка является избыточной, если переменная встречается лишь в одном аргументе лишь одного логарифма. В нашем случае х действительно стоит лишь в аргументе и лишь под одним знаком log. Следовательно, никаких дополнительных проверок выполнять не требуется.

Тем не менее, если вы не доверяете данному методу, то легко можете убедиться, что х = 2 действительно является корнем. Достаточно подставить это число в исходное уравнение.

Давайте перейдем ко второму уравнению, оно чуть интересней:

log2 (log1/2 (2x− 1) + log2 4) = 1

Если обозначить выражение внутри большого логарифма функцией f(x), получим простейшее логарифмическое уравнение, с которого мы начинали сегодняшний видеоурок. Следовательно, можно применить каноническую форму, для чего придется представить единицу в виде log2 21 = log2 2.

Переписываем наше большое уравнение:

log2 (log1/2 (2x − 1) + log2 4) = log2 2

Изваляемся от знака логарифма, приравнивая аргументы. Мы вправе это сделать, потому что и слева, и справа основания одинаковые. Кроме того, заметим, что log2 4 = 2:

log1/2 (2x− 1) + 2 = 2

log1/2 (2x− 1) = 0

Перед нами снова простейшее логарифмическое уравнение вида logaf(x) = b. Переходим к канонической форме, т. е. представляем ноль в виде log1/2 (1/2)0 = log1/2 1.

Переписываем наше уравнение и избавляемся от знака log, приравнивая аргументы:

log1/2 (2x− 1) = log1/2 1

2x − 1 = 1

2х = 2

х = 1

Опять же мы сразу получили ответ. Никаких дополнительных проверок не требуется, потому что в исходном уравнении лишь один логарифм содержит функцию в аргументе.

Следовательно, никаких дополнительных проверок выполнять не требуется. Мы можем смело утверждать, что х = 1 является единственным корнем данного уравнения.

А вот если бы во втором логарифме вместо четверки стояла бы какая-то функция от х (либо 2х стояло бы не в аргументе, а в основании) — вот тогда потребовалось бы проверять область определения. Иначе велик шанс нарваться на лишние корни.

Откуда возникают такие лишние корни? Этот момент нужно очень четко понимать. Взгляните на исходные уравнения: везде функция х стоит под знаком логарифма. Следовательно, поскольку мы записали log2x, то автоматически выставляем требование х > 0. Иначе данная запись просто не имеет смысла.

Однако по мере решения логарифмического уравнения мы избавляемся от всех знаков log и получаем простенькие конструкции. Здесь уже никаких ограничений не выставляется, потому что линейная функция определена при любом значении х.

Именно эта проблема, когда итоговая функция определена везде и всегда, а исходная — отнюдь не везде и не всегда, и является причиной, по которой в решении логарифмических уравнениях очень часто возникают лишние корни.

Но повторю еще раз: такое происходить лишь в ситуации, когда функция стоит либо в нескольких логарифмах, либо в основании одного из них. В тех задачах, которые мы рассматриваем сегодня, проблем с расширением области определения в принципе не существует.

Тонкости и хитрости решения

Сегодня мы переходим к более сложным задачам и будем решать логарифмическое уравнение, в основании которого стоит не число, а функция.

И пусть даже эта функция линейна — в схему решения придется внести небольшие изменения, смысл которых сводится к дополнительным требованиям, накладываемым на область определения логарифма.

Логарифмические уравнения. Методы решения

На самом деле существует целая масса подходов: это и разложение на множители, и потенцирование, и замена, и работа с основаниями…

Но все методы решения логарифмических уравнения роднит одно: их цель свести логарифмические уравнения к простейшему виду::

Если уравнение сведено к такому, что слева и справа от знака «равно» стоят логарифмы с одним основанием, то логарифмы мы «зачеркиваем» и решаем оставшееся уравнение.

Однако, тут есть один подводный камень: поскольку логарифм определен только тогда, когда

то после нахождения корней логарифмического уравнения, мы обязаны сделать проверку!!! Я не поленюсь и повторю еще раз:

В ЛОГАРИФМИЧЕСКИХ УРАВНЕНИЯХ МЫ ВСЕГДА ДЕЛАЕМ ПРОВЕРКУ ПОЛУЧЕННЫХ КОРНЕЙ!!

Те учащиеся, которые игнорируют это требование, как правило допускают глупейшие и непростительные ошибки!

Согласись, обидно решить правильно уравнение, а потом не сделать самую малость: проверку, и записать лишние корни, и записать из-за этого неправильный ответ!

Формулы логарифмов. Логарифмы примеры решения

Теперь на основе этих формул(свойств), покажем примеры решения логарифмов.

Примеры решения логарифмов на основании формул

Логарифм положительного числа b по основанию a (обозначается logab) — это показатель степени, в которую надо возвести a, чтобы получить b, при этом b > 0, a > 0, а 1.

Согласно определения logab = x, что равносильно ax = b, поэтому logaax = x.

Логарифмы, примеры:

log28 = 3, т.к. 23 = 8

log749 = 2, т.к. 72 = 49

log51/5 = -1, т.к. 5-1 = 1/5

Десятичный логарифм — это обычный логарифм, в основании которого находится 10. Обозначается как lg.

lg100 = 2

log10100 = 2, т.к. 102 = 100

Натуральный логарифм — также обычный логарифм логарифм, но уже с основанием е (е = 2,71828… — иррациональное число). Обозначается как ln.

Формулы или свойства логарифмов желательно запомнить, потому что они понадобятся нам в дальнейшем при решении логарифмов, логарифмических уравнений и неравенств. Давайте еще раз отработаем каждую формулу на примерах.

Основное логарифмическое тождество
a logab = b
Пример.
82log83 = (82log83)2 = 32 = 9

Логарифм произведения равен сумме логарифмов loga (bc) = logab + logac
Пример.
log38,1 + log310 = log3 (8,1*10) = log381 = 4

Логарифм частного равен разности логарифмов
loga (b/c) = logab — logac
Пример.
9 log550/9 log52 = 9 log550- log52 = 9 log525 = 9 2 = 81

Свойства степени логарифмируемого числа и основания логарифма

Показатель степени логарифмируемого числа logab m = mlogab

Показатель степени основания логарифма loganb =1/n*logab

loganb m = m/n*logab,

если m = n, получим loganb n = logab

Пример.

log49 = log223 2 = log23

Переход к новому основанию
logab = logcb/logca,

если c = b, получим logbb = 1

тогда logab = 1/logba

Пример.

log0,83*log31,25 = log0,83*log0,81,25/log0,83 = log0,81,25 = log4/55/4 = -1

Как видите, формулы логарифмов не так сложны как кажутся. Теперь рассмотрев примеры решения логарифмов мы можем переходить к логарифмическим уравнениям.

Область допустимых значений (ОДЗ) логарифма

Теперь поговорим об ограничениях (ОДЗ – область допустимых значений переменных).

Мы помним, что, например, квадратный корень нельзя извлекать из отрицательных чисел; или если у нас дробь, то знаменатель не может быть равен нулю. Подобные ограничения есть и у логарифмов:

То есть и аргумент, и основание должны быть больше нуля, а основание еще и не может равняться .

Почему так?

Начнем с простого: допустим, что . Тогда, например, число не существует, так как в какую бы степень мы не возводили , всегда получается . Более того, не существует ни для какого . Но при этом может равняться чему угодно (по той же причине – в любой степени равно ). Поэтому объект не представляет никакого интереса, и его просто выбросили из математики.

Похожая проблема у нас и в случае : в любой положительной степени – это , а в отрицательную его вообще нельзя возводить, так как получится деление на ноль (напомню, что ).

При мы столкнемся с проблемой возведения в дробную степень (которая представляется в виде корня: . Например, (то есть ), а вот не существует.

Поэтому и отрицательные основания проще выбросить, чем возиться с ними.

Ну а поскольку основание a у нас бывает только положительное, то в какую бы степень мы его ни возводили, всегда получим число строго положительное. Значит, аргумент должен быть положительным. Например, не существует, так как ни в какой степени не будет отрицательным числом (и даже нулем, поэтому тоже не существует).

В задачах с логарифмами первым делом нужно записать ОДЗ. Приведу пример:

Решим уравнение .

Вспомним определение: логарифм – это степень, в которую надо возвести основание , чтобы получить аргумент . И по условию, эта степень равна : .

Получаем обычное квадратное уравнение: . Решим его с помощью теоремы Виета: сумма корней равна , а произведение . Легко подобрать, это числа и .

Но если сразу взять и записать оба этих числа в ответе, можно получить 0 баллов за задачу. Почему? Давайте подумаем, что будет, если подставить эти корни в начальное уравнение?

– верно.

– это явно неверно, так как основание не может быть отрицательным, то есть корень – «сторонний».

Чтобы избежать таких неприятных подвохов, нужно записать ОДЗ еще до начала решения уравнения:

Тогда, получив корни и , сразу отбросим корень , и напишем правильный ответ.

Использование свойств логарифмов при вычислении

Мощным инструментом вычисления логарифмов является использование свойств логарифмов.

Некоторые свойства логарифмов позволяют сразу указать значение логарифмов. К таким свойствам относятся свойство логарифма единицы и свойство логарифма числа, равного основанию: log11=logaa0=0 и logaa=logaa1=1. То есть, когда под знаком логарифма находится число 1 или число a, равное основанию логарифма, то в этих случаях логарифмы равны 0 и 1 соответственно.

Чему равны логарифмы и lg10?

Так как , то из определения логарифма следует .

Во втором примере число 10 под знаком логарифма совпадает с его основанием, поэтому десятичный логарифм десяти равен единице, то есть, lg10=lg101=1.

и lg10=1.

Отметим, что вычисление логарифмов по определению (которое мы разобрали в предыдущем пункте) подразумевает использование равенства logaap=p, которое является одним из свойств логарифмов.

На практике, когда число под знаком логарифма и основание логарифма легко представляются в виде степени некоторого числа, очень удобно использовать формулу , которая соответствует одному из свойств логарифмов. Рассмотрим пример нахождения логарифма, иллюстрирующий использование этой формулы.

Вычислите логарифм .

.

Не упомянутые выше свойства логарифмов также используются при вычислении, но об этом поговорим в следующих пунктах.

Зачем в жизни нужны логарифмы?

Я уже говорил, что математики СУПЕРленивые люди? Это правда.

Вот представь себе, им лень умножать и они придумали логарифмы, которые позволяют заменить умножение сложением!

Им еще больше лень возводить в степень и они используют логарифмы, чтобы заменить возведение в степень умножением или делением!

То есть они используют логарифмы, чтобы быстро проделывать громоздкие вычисления.

Круто, да?

Пример Найдите корень уравнения.

Здесь для решения данного логарифмического уравнения будем использовать свойство логарифма:

То есть внесем число 3 справа под знак логарифма.

или

Если показатели степени равны, основания степени равны, то равны числа, получаемые в результате, то есть получим

Делаем проверку:

Получаем:

Ответ:

Степень можно выносить за знак логарифма

log a b p =p log a b (a>0,a≠1,b>0) (7)

И вновь хотелось бы призвать к аккуратности. Рассмотрим следующий пример:

log a (f (x) 2 =2 log a f(x)

Левая часть равенства определена, очевидно, при всех значениях f(х), кроме нуля. Правая часть — только при f(x)>0! Вынося степень из логарифма, мы вновь сужаем ОДЗ. Обратная процедура приводит к расширению области допустимых значений. Все эти замечания относятся не только к степени 2, но и к любой четной степени.

Формула перехода к новому основанию

log a b= log c b log c a (a>0,a≠1,b>0,c>0,c≠1) (8)

Тот редкий случай, когда ОДЗ не изменяется при преобразовании. Если вы разумно выбрали основание с (положительное и не равное 1), формула перехода к новому основанию является абсолютно безопасной.

Если в качестве нового основания с выбрать число b, получим важный частный случай формулы (8):

log a b= 1 log b a (a>0,a≠1,b>0,b≠1) (9)

Десятичным логарифмом числа x называется логарифм по основанию 10. Десятичные логарифмы используются довольно часто, поэтому для них введено специальное обозначение: log10x = lg x. Все перечисленные выше формулы сохраняют актуальность для десятичных логарифмов. Например, lg(xy)=lgx+lgy (x>0,y>0) .

Натуральным логарифмом числа x (обозначение lnx) называется логарифм х по основанию e. Число e — иррациональное, приближенно равно 2,71. Например, ln e = 1. Пользуясь формулой (8), можно любой логарифм свести к десятичным или натуральным логарифмам: log a b= lgb lga = lnb lna (a>0,a≠1,b>0)

Несколько простых примеров с логарифмами

Пример 1. Вычислите: lg2 + lg50. Решение. lg2 + lg50 = lg100 = 2. Мы воспользовались формулой суммы логарифмов (5) и определением десятичного логарифма.

Пример 2. Вычислите: lg125/lg5. Решение. lg125/lg5 = log5125 = 3. Мы использовали формулу перехода к новому основанию (8).

a log a b =b (a>0,a≠1)
log a a=1 (a>0,a≠1)
log a 1=0 (a>0,a≠1)
log a (bc)= log a b+ log a c (a>0,a≠1,b>0,c>0)
log a b c = log a b− log a c (a>0,a≠1,b>0,c>0)
log a b p =p log a b (a>0,a≠1,b>0)
log a b= log c b log c a (a>0,a≠1,b>0,c>0,c≠1)
log a b= 1 log b a (a>0,a≠1,b>0,b≠1)

Сложные задачи

Этот урок будет довольно длинным. В нем мы разберем два довольно серьезных логарифмических уравнения, при решении которых многие ученики допускают ошибки. За свою практику работы репетитором по математике я постоянно сталкивался с двумя видами ошибок:

  1. Возникновение лишних корней из-за расширения области определения логарифмов. Чтобы не допускать такие обидные ошибки, просто внимательно следите за каждым преобразованием;
  2. Потери корней из-за того, что ученик забыл рассмотреть некоторые «тонкие» случаи — именно на таких ситуациях мы сегодня и сосредоточимся.

Это последний урок, посвященный логарифмическим уравнениям. Он будет длинным, мы разберем сложные логарифмические уравнения. Устраивайтесь поудобней, заварите себе чай, и мы начинаем.

Первое уравнение выглядит вполне стандартно:

logx + 1 (x − 0,5) = logx − 0,5 (x + 1)

Сразу заметим, что оба логарифма являются перевернутыми копиями друг друга. Вспоминаем замечательную формулу:

logab = 1/logba

Однако у этой формулы есть ряд ограничений, которые возникают в том случае, если вместо чисел а и b стоят функции от переменной х:

b > 0

1 ≠ a > 0

Эти требования накладываются на основание логарифма. С другой стороны, в дроби от нас требуется 1 ≠ a > 0, поскольку не только переменная a стоит в аргументе логарифма ( следовательно, a > 0), но и сам логарифм находится в знаменателе дроби. Но logb 1 = 0, а знаменатель должен быть отличным от нуля, поэтому a ≠ 1.

Итак, ограничения на переменную a сохраняется. Но что происходит с переменной b? С одной стороны, из основания следует b> 0, с другой — переменная b≠ 1, потому что основание логарифма должно быть отлично от 1. Итого из правой части формулы следует, что 1 ≠ b > 0.

Но вот беда: второе требование (b ≠ 1) отсутствует в первом неравенстве, посвященном левому логарифму. Другими словами, при выполнении данного преобразования мы должны отдельно проверить, что аргумент bотличен от единицы!

Вот давайте и проверим. Применим нашу формулу:

[Подпись к рисунку]

А теперь, прежде чем идти дальше, выпишем все требования области определения, накладываемые на исходную задачу:

1 ≠ х − 0,5 > 0; 1 ≠ х + 1 > 0

Вот мы и получили, что уже из исходного логарифмического уравнения следует, что и а, и b должны быть больше 0 и не равны 1. Значит, мы спокойно можем переворачивать логарифмическое уравнение:

Предлагаю ввести новую переменную:

logx + 1 (x − 0,5) = t

В этом случае наша конструкция перепишется следующим образом:

(t2− 1)/t = 0

Заметим, что в числителе у нас стоит разность квадратов. Раскрываем разность квадратов по формуле сокращенного умножения:

(t − 1)(t + 1)/t = 0

Дробь равна нулю, когда ее числитель равен нулю, а знаменатель отличен от нуля. Но в числителе стоит произведение, поэтому приравниваем к нулю каждый множитель:

t1 = 1;

t2 = −1;

t ≠ 0.

Как видим, оба значения переменной tнас устраивают. Однако на этом решение не заканчивается, ведь нам требуется найти не t, а значение x. Возвращаемся к логарифму и получаем:

logx + 1 (x − 0,5) = 1;

logx + 1 (x − 0,5) = −1.

Давайте приведем каждое из этих уравнений к канонической форме:

logx + 1 (x − 0,5) = logx + 1 (x + 1)1

logx + 1 (x − 0,5) = logx + 1 (x + 1)−1

Избавляемся от знака логарифма в первом случае и приравниваем аргументы:

х − 0,5 = х + 1;

х − х = 1 + 0,5;

0 = 1,5.

Такое уравнение не имеет корней, следовательно, первое логарифмическое уравнение также не имеет корней. А вот со вторым уравнением все намного интересней:

(х − 0,5)/1 = 1/(х + 1)

Решаем пропорцию — получим:

(х − 0,5)(х + 1) = 1

Напоминаю, что при решении логарифмических уравнений гораздо удобней приводить все десятичные дроби обычные, поэтому давайте перепишем наше уравнение следующим образом:

(х − 1/2)(х + 1) = 1;

x2 + x− 1/2x− 1/2 − 1 = 0;

x2 + 1/2x− 3/2 = 0.

Перед нами приведенное квадратное уравнение, оно легко решается по формулам Виета:

(х + 3/2) (х − 1) = 0;

x1 = −1,5;

x2 = 1.

Получили два корня — они являются кандидатами на решение исходного логарифмического уравнения. Для того чтобы понять, какие корни действительно пойдут в ответ, давайте вернемся к исходной задаче. Сейчас мы проверим каждый из наших корней на предмет соответствия области определения:

1,5 ≠ х > 0,5; 0 ≠ х > −1.

Эти требования равносильны двойному неравенству:

1 ≠ х > 0,5

Отсюда сразу видим, что корень х = −1,5 нас не устраивает, а вот х = 1 вполне устраивает. Поэтому х = 1 — окончательное решение логарифмического уравнения.

Переходим ко второй задаче:

logx 25 + log125x 5 = log25x 625

На первый взгляд может показаться, что у всех логарифмов разные основания и разные аргументы. Что делать с такими конструкциями? В первую очередь заметим, что числа 25, 5 и 625 — это степени 5:

25 = 52>;>4

А теперь воспользуемся замечательным свойством логарифма. Дело в том, что можно выносить степени из аргумента в виде множителей:

logabn = n ∙ logab

На данное преобразование также накладываются ограничения в том случае, когда на месте bстоит функция. Но у нас b— это просто число, и никаких дополнительных ограничений не возникает. Перепишем наше уравнение:

2 ∙ logx 5 + log125x 5 = 4 ∙ log25x 5

Получили уравнение с тремя слагаемыми, содержащими знак log. Причем аргументы всех трех логарифмов равны.

Самое время перевернуть логарифмы, чтобы привести их к одному основанию — 5. Поскольку в роли переменной b выступает константа, никаких изменений области определения не возникает. Просто переписываем:

Как и предполагалось, в знаменателе «вылезли» одни и те же логарифмы. Предлагаю выполнить замену переменной:

log5x = t

В этом случае наше уравнение будет переписано следующим образом:

Выпишем числитель и раскроем скобки:

2 (t + 3) (t + 2) + t (t + 2) − 4t (t + 3) = 2 (t2 + 5t + 6) + t2 + 2t − 4t2 − 12t = 2t2 + 10t + 12 + t2 + 2t − 4t2 − 12t = −t2 + 12

Возвращаемся к нашей дроби. Числитель должен быть равен нулю:

А знаменатель — отличен от нуля:

t ≠ 0; t ≠ −3; t ≠ −2

Последние требования выполняются автоматически, поскольку все они «завязаны» на целые числа, а все ответы — иррациональные.

Итак, дробно-рациональное уравнение решено, значения переменной t найдены. Возвращаемся к решению логарифмического уравнения и вспоминаем, что такое t:

Приводим это уравнение к канонической форме, получим число с иррациональной степенью. Пусть это вас не смущает — даже такие аргументы можно приравнять:

У нас получилось два корня. Точнее, два кандидата в ответы — проверим их на соответствие области определения. Поскольку в основании логарифма стоит переменная х, потребуем следующее:

1 ≠ х > 0;

С тем же успехом утверждаем, что х ≠ 1/125, иначе основание второго логарифма обратится в единицу. Наконец, х ≠ 1/25 для третьего логарифма.

Итого мы получили четыре ограничения:

1 ≠ х > 0; х ≠ 1/125; х ≠ 1/25

А теперь вопрос: удовлетворяют ли наши корни указанным требованиям? Конечно удовлетворяют! Потому что 5 в любой степени будет больше нуля, и требование х > 0 выполняется автоматически.

С другой стороны, 1 = 50, 1/25 = 5−2, 1/125 = 5−3, а это значит, что данные ограничения для наших корней (у которых, напомню, в показателе стоит иррациональное число) также выполнены, и оба ответа являются решениями задачи.

Итак, мы получили окончательный ответ. Ключевых моментов в данной задаче два:

  1. Будьте внимательны при перевороте логарифма, когда аргумент и основание меняются местами. Подобные преобразования накладывают лишние ограничения на область определения.
  2. Не бойтесь преобразовывать логарифмы: их можно не только переворачивать, но и раскрывать по формуле суммы и вообще менять по любым формулам, которые вы изучали при решении логарифмических выражений. Однако при этом всегда помните: некоторые преобразования расширяют область определения, а некоторые — сужают.

Логарифм: что это? Все формулы. Простейшие уравнения и неравенства

Сейчас речь пойдет о трех страшных буквах: l o g.Существовать в нашем бытии они просто так не могут. Обязательно должен быть какой-нибудь индекс — число снизу (основание логарифма) и число после букв (аргумент логарифма).

Прежде, чем мы перейдем к тому, что такое логарифм, решим парочку подводящих примеров.

Чтобы справиться с этим примером, мы проговариваем в голове: какое число нужно дважды (т.к. корень квадратный) умножить само на себя, чтобы получить 81.

А этот пример можно решить по алгоритму (решения показательных уравнений), а можно так же провести разговор с самим собой (главное не вслух, я считаю это нормально, но кого-то вы можете напугать разговором с самим собой): сколько раз нужно число 3 умножить само на себя, чтобы получить 27. Постепенным перемножением мы дойдем до ответа.

Тогда, если дело касается логарифма:

можно сказать так: в какую степень нужно возвести 3 (число снизу — основание логарифма), чтобы получить 27 (число слева — аргумент логарифма). Не напоминает выше стоящий пример?

На самом деле в этом и заключается основная формула (определение логарифма):

Логарифм говорит нам (кому-то кричит): логарифм числа «b» по основанию «a» равняется числу «c». Тогда без логарифма это можно сформулировать так: чтобы получить число «b», требуется число «a» возвести в степень «c». Логарифм — это действие, обратное возведению в степень.

У отца log есть два родных сына: ln и lg. Так же, как сыновья отличаются возрастом (мы говорим о максимальной точности), так и эти логарифмы отличаются основанием (числовым индексом снизу).

Данные логарифмы придумали для упрощения записи. На самом деле в прикладной математики именно логарифмы по такому основанию встречаются чаще всех остальных. А мы все в глубине души народ ленивый, так что почему бы себе жизнь не упростить?

Что нужно запомнить: ln — это обычный логарифм только по основанию e ( e — это число Эйлера, e = 2,7182…, мой номер телефона, кстати, — это последние 11 цифр числа Эйлера, так что буду ждать звонка).

А lg — это обычный логарифм по основанию 10 (10ая система — это система счисления, в которой мы живем, столько пальцев на руках у среднего человека. В общем 10 — это как 9, только на 1 больше).

Как мы не можем существовать без еды, воды, интернета… Так и логарифм не представляет свое существование без ОДЗ.

Всегда, когда существует логарифм, должно быть:

«Почему это так?» — это первый вопрос, который я предоставляю тебе. Советую начать с того, что логарифм — это обратное действие от возведения в степень.

А теперь разберем теорию на практике:

В какую степень нужно возвести два (число в основании), чтобы получить шестнадцать (аргумент логарифма).

Два нужно четыре раза умножить само на себя, чтобы получить 16.

Ответ: 4.

Нахождение логарифмов через другие известные логарифмы

Информация этого пункта продолжает тему использования свойств логарифмов при их вычислении. Но здесь основное отличие состоит в том, что свойства логарифмов используются для того, чтобы выразить исходный логарифм через другой логарифм, значение которого известно. Приведем пример для пояснения. Допустим, мы знаем, что log23≈1,584963, тогда мы можем найти, например, log26, выполнив небольшое преобразование с помощью свойств логарифма: log26=log2(2·3)=log22+log23≈ 1+1,584963=2,584963.

В приведенном примере нам было достаточно использовать свойство логарифма произведения. Однако намного чаще приходится применять более широкий арсенал свойств логарифмов, чтобы вычислить исходный логарифм через заданные.

Вычислите логарифм 27 по основанию 60, если известно, что log602=a и log605=b.

Итак, нам нужно найти log6027. Несложно заметить, что 27=33, и исходный логарифм в силу свойства логарифма степени можно переписать как 3·log603.

Теперь посмотрим, как log603 выразить через известные логарифмы. Свойство логарифма числа, равного основанию, позволяет записать равенство log6060=1. С другой стороны log6060=log60(22·3·5)=log6022+log603+log605=2·log602+log603+log605. Таким образом, 2·log602+log603+log605=1. Следовательно, log603=1−2·log602−log605=1−2·a−b.

Наконец, вычисляем исходный логарифм: log6027=3·log603=3·(1−2·a−b)=3−6·a−3·b.

log6027=3·(1−2·a−b)=3−6·a−3·b.

Отдельно стоит сказать о значении формулы перехода к новому основанию логарифма вида . Она позволяет от логарифмов с любыми основаниями переходить к логарифмам с конкретным основанием, значения которых известны или есть возможность их отыскать. Обычно от исходного логарифма по формуле перехода переходят к логарифмам по одному из оснований 2, e или 10, так как по этим основаниям существуют таблицы логарифмов, позволяющие с определенной степенью точности вычислять их значения. В следующем пункте мы покажем, как это делается.

Источники

  • http://www.cleverstudents.ru/logarithms/computation_of_logarithms.html
  • https://www.berdov.com/docs/logarithm/reshenie-logarifmicheskih-uravneniy/
  • https://youclever.org/book/logarifmicheskie-uravneniya-1
  • https://rgiufa.ru/matematika-fizika-himiya/kakie-sushhestvuyut-formuly-logarifmov.html
  • https://youclever.org/book/logarifmy-1
  • https://repetitor-mathematics.ru/kak-reshat-logarifmicheskie-uravneniya-podrobnyiy-razbor/

Вычисление логарифмов

Математика – это ключ и дверь ко всем наукам. 

Великий итальянский ученый  Галилео Галилей

К числу типовых задач, предлагаемых на вступительных испытаниях, являются  задачи, связанные с вычислением логарифмов. Для успешного решения таких задач необходимо знать свойства логарифмов и иметь навыки решения соответствующих задач.

В настоящей статье сначала приводятся основные понятия и свойства, используемые при решении таких задач, а затем рассматриваются примеры решения задач на вычисление логарифмов.

Основные понятия и свойства

Появлению логарифмов человечество обязано шотландскому математику Джону Неперу (1550 – 1617), который 1614 году опубликовал сочинение «Описание удивительной таблицы логарифмов».

Первоначально приведем основные свойства логарифмов, использование которых позволяет вычислять довольно-таки сложные логарифмы из заданий конкурсных испытаний по математике.

Основное логарифмическое тождество записывается в виде

,               (1)

где  ,    и  .

К числу наиболее известных свойств логарифмов относятся следующие равенства:

1. Если , ,  и  , то   ,  ,

,  .

2. Если , , ,  и  , то   .

3. Если  , ,  и  , то   .

4. Если   , ,  и  натуральное число,  то  

.

5. Если   , ,  и  натуральное число,  то  

6. Если   , ,    и   ,  то  .

7. Если   , ,    и   ,  то  .

Более сложные свойства логарифмов формулируются посредством следующих утверждений.

8.  Если  , , ,  и  , то  

.

9.  Если  , ,  и  , то  

.   

10.  Если  , , ,  и  , то  

.

    Доказательство последних двух свойств логарифмов приведено в учебном пособии автора «Математика для старшеклассников: дополнительные разделы школьной математики» (М.: Ленанд / URSS, 2014).  

    Если  , то логарифм    называется «десятичным» и обозначается . Если  , где  натуральная константа, введенная Леонардом Эйлером (1707 – 1783), то логарифм имеет название «натуральный» и он обозначается  .

    Отметим, что в настоящее время имеются различные таблицы десятичных и натуральных логарифмов, которые широко используются в инженерных расчетах.

Рассмотрим примеры задач на вычисление логарифмов, решение которых основано на применении приведенных выше свойств.

Примеры решения задач

Пример 1.   Вычислить  .

Решение.  Обозначим    и  .  Так как  

   и    ,  то

  и   .

Поскольку  ,  и  , то  .  

Ответ:  .

Пример 2.   Вычислить  .

Решение. Так как  , где   и , то

.

Ответ:  .

Пример 3. Вычислить  .

Решение. Используя формулу (1), получаем . В этой связи   .

Ответ:  .

Пример 4. Вычислить  .

Решение.  Так как      и   ,  то

.

Ответ:  .

Пример 5.   Вычислить  .

Решение. Поскольку  , то    или  .

Ответ:  .

Пример 6. Вычислить  .

Решение. Так как   и  ,  то

.

Ответ:  .

Пример 7. Вычислить  .

Решение. Так как   и  ,  то

.

Ответ:  .

Пример 8.  Вычислить  .

Решение. Используя свойства 2 и 4, можно записать

.

Ответ:   .

Пример 9.  Вычислить  . 

Решение.  Принимая во внимание свойство 8, здесь имеем

.

Поскольку  , то отсюда получаем  .

Ответ:  .

Пример 10.  Вычислить  .

Решение.  Здесь, используя свойства 1 и 2, получаем

.

Ответ:  .

Пример 11.  Вычислить  .

Решение.  Используя свойства 4 и 5, а также формулу (1), можно записать  

  и   

.

Поскольку  , то  .  В этой связи имеем

.

Ответ:  .

Пример 12.  Доказать, что   .

Доказательство. Обозначим . Если преобразовать    на основе двукратного применения свойства 8, то получим  .

Равенство доказано.

Пример 13. Вычислить  . 

Решение.  Используя свойство 3, получаем

.               (2)

Так как     ,    и   

,  то выражение (2) принимает вид

.

Ответ:  .

Пример 14.   Упростить  .

Решение.  Так как    и   , то  

  или  .

Тогда       .

Ответ:  .

Пример 15.  Вычислить  . 

Решение. Согласно свойству 10 имеем . Далее, из свойства 9 следует, что  .  В этой связи

.

Следовательно, окончательно получаем  .

Ответ:  .

Пример 16.  Вычислить    при условии, что

.

Решение.  Так как , то   или  . Преобразуем выражение на основе использования свойств 1 и 2, следующим образом:

 или   .

Так как  , то отсюда получаем   .

Ответ:  .

Пример 17.  Вычислить  , если    и  .

Решение.  Так как    и  

,  то  .

Ответ:  .

Пример 18.  Дано: и . Выразить посредством функции, зависящей от переменной  .

Решение.  Используя свойства 1 и 2, представим    и   посредством следующих формул:

,               (3)

.               (4)

Так как обе формулы содержат  , то для решения задачи необходимо выразить    из формулы (4) и полученное выражение подставить в формулу (3). Поскольку         

   или   ,   то

.

Ответ:  

Пример 19.  Доказать, что   .

Доказательство.  Обозначим  .  Преобразуем    следующим образом:  .  Так как  , то . Далее воспользуемся неравенством Коши  или  , которое выполняется для любых    и  .

В таком случае  .

Неравенство доказано.

Примечание. При доказательстве неравенства использовалось свойство логарифмической функции: если , то функции является возрастающей на всей области определения.

Пример 20.  Дано: , ,  ,  и . Вычислить  .

Решение.  Так как  , то согласно свойству 3 имеем

  или   .  

Так как  , ,    и  ,  то

,   ,  ,    и   .

Ответ:  .

Для более глубокого изучения методов решения задач на вычисление логарифмов целесообразно обратиться к учебным пособиям, приведенных в списке рекомендованной литературы.  

   

Рекомендуемая литература

1.  Кушнир А.И. Шедевры школьной математики (задачи и решения в двух книгах). – Киев: Астарта, книга 1, 1995. – 576 с.

2. Сборник задач по математике для поступающих во втузы / Под ред. М.И. Сканави. – М.: Мир и Образование, 2013. – 608 с.

3. Супрун В.П. Математика для старшеклассников: дополнительные разделы школьной программы. – М.: Ленанд / URSS, 2014. – 216 с.

Остались вопросы? 

Чтобы получить помощь репетитора – зарегистрируйтесь.

© blog.tutoronline.ru,
при полном или частичном копировании материала ссылка на первоисточник обязательна.

Алгебра и начала анализа в 11-м классе. Тема: «Логарифмы и его свойства»

Цели:

  • Методическая: Повышение
    активно-познавательной деятельности учащихся
    путем проведения индивидуально-самостоятельной
    работы и применения опережающих заданий.
  • Обучающая: Повторение определения
    показательной функции, основные свойства
    степеней. Ввести понятие логарифма и его свойств.
    Решение упражнений. Систематизировать знания
    учащихся по теме; способствовать выработке
    умений и навыков в вычислении логарифмов, в
    применении их свойств при логарифмировании и
    потенцировании; рассмотреть более сложные
    примеры по теме и проверить навыки и умения при
    самостоятельном решении упражнений.
  • Воспитательная: Воспитание аккуратности,
    собранности. Проверить сформированность качеств
    знаний: прочность, глубина, оперативность;
    формировать гуманные отношения на уроке через
    работу в парах, умение слушать друг друга;
    добросовестное отношение к учебному труду,
    ответственность, честность, сопереживание
    успехам и неудачам товарищей.
  • Развивающая: Развивать интеллектуальные
    способности, мыслительные процессы, речь, память.
    Развивать любовь и интерес к математике.
    Установить, могут ли студенты применять знания
    логарифмов при решении задач; в ходе урока
    обеспечить развитие у учащихся
    самостоятельности мышления и в учебной
    деятельности;

Тип урока: Комбинированный

Форма проведения урока: Индивидуальная
и фронтальная. Решение примеров.

Оборудование: Плакаты на тему “
Показательная функция”, “Логарифмы и его
свойства”, “гармошка” с заданиями, таблицы.
Карточки-задания для индивидуальной
самостоятельной работы, лото-задания, учебная
литература.

Эпиграф урока:



“Математику уже затем учить надо, что
она ум в порядок приводит”

(М.В. Ломоносов)


 Ход урока



1. Организационный момент

Вступительное слово преподавателя.

Я приветствую вас на сегодняшнем уроке алгебры.
Тема урока: “Логарифм и его свойства”. Сегодня
мы повторим понятие логарифма числа, свойства
логарифма, закрепим умения применять эти понятия
при решении уравнений. Эпиграфом урока являются
слова М.В. Ломоносова

“Математику уже затем учить надо, что она ум в
порядок приводит”. На доске: дата, тема, план,
эпиграф урока.

Объявляется цель и задачи урока. Раздаются
тетради для самостоятельных работ и учебники.

2. Актуализация опорных знаний, умений
студентов

1. Проверка домашнего задания. №472а), №473а).

(Вызвать 2-х учащихся к доске.)

№ 272а)

>2,5; > 2,5; > ; > 1; ;

Х <0.

Ответ: х .

№ 273а).

2х> ; х2 – 3 + 2х
> 0; х2 + 2х – 3 > 0.

Решим методом интервалов.

Пусть f (x) = х2 + 2х – 3, где f (x) = 0, т. е. х2
+ 2х – 3 = 0.

Д = 22 – 4 * (- 3) = 4 = 12 = 16.

х1 = х2
=

f (0) = 02 + 2 * 0 – 3 < 0.

Ответ: х (-;-3)U(1;+ )



2. Индивидуальная работа по карточкам (10 мин). (Приложение 1)

(Раздать 6 карточек в виде лото-задания.)



Задания:

  1. Решить показательные уравнения.
  2. Решить показательные неравенства.
  3. Задания на свойства показательной функции.



3. Индивидуальная работа у доски.

(Написать свойства степеней.)



4. Выступление у доски.

(Доклад о происхождении степеней. Сведения из
истории) Таблички с именами ученых готовить
заранее. (Приложение 2)

5. По стенду дать определение показательной
функции, свойства показательной функции.

Далее проверить домашнее задание. (Учащиеся
комментирует план выполнения работ.)

Ассистенты самостоятельно проверяют
индивидуальные работы учащихся, работающих
индивидуально.



6. По “гармошке” решить показательные
неравенства и уравнения ( задания готовить
заранее.)

(Приложение 3)

3. Формирование новых знаний и понятий

Дата, тема и план урока написать заранее.
Эпиграф урока на доске.

Преподаватель: Вы знакомы с шестью
действиями над числами



А+ВА-ВА*ВА/ВА

Эти действия образуют три пары взаимно
обратных действий. А для того чтобы решить
уравнение а=в
, где а>0 и а 1 придумали
седьмое действие, которое называется логарифмом.

logа в=с ,где b>0 , а>0, а1, в=а

Преподаватель:



Совершаем небольшой экскурс в историю
математики. Ученики слушают сообщение на тему
“Изобретение логарифма”. На доске записи,
которые предлагаем записать в тетрадь.

Джон Непер – 1614 год – изобретение логарифма

Бригс – 1624 год – создание таблиц логарифмов.

1703 год – перевод таблиц на русский язык

Л. Магницкий – 1716 год – издание семизначных
логарифмических таблиц.

Логарифмическая линейка

Слово логарифм происходит от греческого (число)
и (отношение) и переводится, следовательно, как
отношение чисел. Выбор изобретателем (1594 г.)
логарифмов Дж. Непером такого названия
объясняется тем, что логарифмы возникли при
сопоставлении двух чисел, одно из которых
является членом арифметической прогрессии, а
другое — геометрической.

Определение. Логарифмом положительного
числа в при положительном
основание а, называется показатель
степени, в которую нужно возвести основание а,
чтобы получить логарифмируемое число в.



Формула
(где в>0, а>0 и а1 )
называется основным логарифмическим
тождеством.



Пример:

1) log9=2 т.к. 9>0,3>0, 3=1, 3 =9.

2) log3=0,5 т.к. 3>0, 9>0, 9=1, 9 =3

Преподаватель: Также существуют
десятичные и натуральные логарифмы.


Log10 x= lg x –это десятичный логарифм.

Logе x=ln x-это натуральный логарифм.




(Таблички с формулами готовить заранее.)

Логарифмы с основанием е ввел С п е й д е л (1619 г.),
составивший первые таблицы для функции In x.
Название более позднего происхождения
натуральный (естественный) объясняется
“естественностью” этого логарифма.

Н. Мерка т о р (1620–1687), предложивший это
название, обнаружил, что In х – это площадь под
гиперболой у = . Он предлагал также название
“гиперболический”.

На этом уроке вы познакомитесь со свойствами
логарифмов, позволяющими преобразовывать
логарифмические выражения, решать
логарифмические уравнения и неравенства.

Для этого следует вспомнить свойства степеней.

К доске вызвать учащегося для записи свойств
степеней.



Свойство 1. logа x=0

Свойство 2. Logа а=1

Свойство 3. Логарифм произведения
положительных чисел равен сумме логарифмов
множителей logа xy= logа x+ logв у

Свойство 4. Логарифм частного двух
положительных чисел равен разности логарифмов
делимого и делителя. logа = logах- logа
у



Свойство 5. Логарифм степени равен
произведению показателя степени на логарифм
основания этой степени. logа x=р logа x

Опережающие задание в виде
сообщения о возникновение логарифмов.

(Имена ученых-математиков заранее готовится на
плакате) (Приложение 4)

4. Применение знаний, навыков, понятий

Работа учащихся у доски.

(Решение примеров №479 а),б), №480 а),б).)

№479 а),б), №480 а),б).- работать самостоятельно.

№ 479

а) log3 =
– 4, так как 3 – 4 = ()4 = ;

б) log 16 1 = 0. Так как 160 = 1.

№ 480.

а) log5 0,04 = -2, т. к. 5-2 = = = 0,04;

б) log7 343 = 3, т. к 73 = 343.

№483 а),б) с объяснением

№ 483.

Найдите логарифмы:

№484



а) log3 х = -1;б) log х = -3;
х = 3-1;х = =216; х =.

№484 а)б), №486 а)б) с объяснением.

№ 486

Опережающие задание в виде сообщения. Тема
“Логарифм и музыка” (Приложение
5
)

(Играет музыка )

Преподаватель: Ребята, логарифмы
применяются на уроках физики. Закон
радиоактивного распада имеет вид m=mе.Формула Циолковского, связывающая
скорость ракеты с ее массой v=v ln .

(Все формулы написаны заранее на листе бумаги
и прикрепляются на доску с помощью магнитиков.)

№488 – №490 решать устно с места (самостоятельная
работа с места по учебнику)

№491 а) Прологарифмируйте по основанию 3 (К доске
вызвать учащегося). Решение комментировать.

№ 491а) log3 = log3 = log3 а3b = (log3 а3 + log3 b) = (3log3 а + log3
b) = log3 а
+ log3 b.
б) log3 =
log3 = (log3 b – log3 a10)
= log3 b –
10 log3 a = log3 b- 2 log3 a.

№492 (а, б) Самостоятельно у доски работает
учащийся.

№ 492.

а) lg 100 = lg 100 +
lg = 2 + lg (ab3c) = 2 + lg ab3c = 2 + (lg a + 3 lg b + lg c)/

Опережающие задание в виде сообщения. Тема
“Звезды, шум и логарифмы” (Приложение
6
)

Преподаватель: Более того,
коэффициент звукоизоляции стен измеряется также
с помощью логарифма, по формуле D=A lg .(Приложение
7
)

№495 (а, б) Самостоятельно у доски. (по времени.)

а) lg 8+ lg 125= lg 8*125= lg 1000=3;

б) log2 7- log2= log2 7/= log2 16=4

– Давайте, ребята, послушаем еще одно
сообщение, которое явно показывает связь
математики и физики (Приложение 8)


5. Обобщение и систематизация знаний
учащихся

1. Словарная работа

– логарифм;

– показательная функция;

– возрастает;

– степень;

– иррациональный показатель;

– тождество.

– экспонента.



2) Эстафета (МАТЕМАТИЧЕСКОЕ ЛОТО.) (Приложение 9)

(заранее готовить 2 варианта заданий.)


6. Итог урока

Подвести итог урока. Сообщить учащимся
оценки, отметить наиболее активных..

7. Домашняя работа

№ 479г) -493г) Учить п.37. Готовится к
математическому диктанту по пройденной теме.

Производные логарифмов и логарифмическое дифференцирование

Что можно сказать о производной логарифмической функции y = lnx на основании
таблицы производных? Можно сказать, что она существует и выражается формулой

                    (1)

Однако в большинстве задач математического анализа, с которыми придётся столкнуться в дальнейшем,
присутствует сложная логарифмическая функция. Она вычисляется несколько иначе.

В случае сложной логарифмической функции y = lnu, где u – дифференцируемая функция аргумента x, формула (1) примет вид

               (2)

Пользуясь формулой (2), найдём производную логарифмической функции с произвольным положительным основанием a. Пусть

В результате применения свойств логарифмов:

Так как

— постоянный множитель, то

или

                    (3)


Если функция дана в виде

,

то перед тем, как находить её производную, часто бывает выгодно прологарифмировать эту функцию.

Это прежде всего случаи, когда требуется найти производную произведения или частного функций,
а также степенной функции, когда основание и степень — функции.

На основании свойств сложных функций доказано, что производная функции, вид которой приведён
выше, может быть найдена по формуле

.

Пример 3. Найти производную функции

.

Решение. Логарифмируем обе части равенства и находим:

Решение. Окончательно находим производную данной функции:

Пример 4. Найти производную функции

.

Решение. Логарифмируем обе части равенства:

Дифференцируем:

Выражаем и находим производную данной функции:

Поделиться с друзьями

Весь блок «Производная»

Изучение логарифмов в старшей школе

Понятие логарифма


При решении показательных уравнений удается представить обе части уравнения в виде степеней с одинаковыми основаниями и рациональными показателями. Так, например, при решении уравнения мы заменяем степенью и из равенства степеней с одинаковыми основаниями делаем вывод о равенстве показателей: х = −5/6. Однако, чтобы решить, казалось бы, более простое уравнение 2х = 3, стандартных знаний оказывается недостаточно. Дело в том, что число 3 нельзя представить в виде степени с основанием 2 и рациональным показателем.


Действительно, если бы равенство , где m и n — натуральные числа, было верным, то, возведя его в степень n, мы должны были бы получить верное равенство 2m = 3n. Но последнее равенство неверно, так как левая его часть является четным числом, а правая — нечетным. Значит, не может быть верным и равенство .


С другой стороны, график непрерывной функции y = 2x пересекается с прямой y = 3, и, значит, уравнение 2x = 3 имеет корень. Таким образом, перед нами стоят два вопроса: «Как записать этот корень?» и «Как его вычислить?».


Показатель степени, в которую нужно возвести число a (a > 0, a ≠ 1), чтобы получить число b, называется логарифмом b по основанию a и обозначается logab.


Теперь мы можем записать корень уравнения 2х = 3:


х = loga3


Равенства ax = b и x = logab, в которых число a положительно и не равно единице, число b положительно, а число x может быть любым, выражают одно и то же соотношение между числами a, b и x. Подставив в первое равенство выражение x из второго, получим основное логарифмическое тождество.

Понятие логарифма в методическом пособии

Задание


Решите уравнение: а) 2x = 64; б) ; в) ; г) 4x = 0; д) 7x = −12.


После проверки ученикам предлагается ответить на вопрос, какое из заданий показалось им наиболее трудным. Вероятный ответ: 2 (в), так как в нем нужно было приводить дробь к степени числа 5. Затем школьникам предлагается высказать мнение о сравнительной с заданием 2 (в) трудности уравнения 2x = 3. На первый взгляд кажется, что это уравнение проще, однако представить 3 в виде степени числа 2 школьникам не удается.


Дальше изучение нового материала проводится в соответствии с учебником. При этом в зависимости от уровня класса рассматривается или не рассматривается дополнительный материал о невозможности представления 3 в виде 2r , где r = m/n.


После этого диалог с классом можно строить примерно так:

— Как вы думаете, имеет ли уравнение 2x = 3 корень? Ответ обоснуйте. [Если построить график функции у = 2x и провести прямую у = 3, то они пересекутся в одной точке, значит, уравнение имеет один корень.]
— Что можно сказать о корне уравнения ax = b, где а > 0 и а ≠ 1? При всех ли значениях b оно имеет корни?


Затем вводится определение логарифма числа b по основанию а и записывается основное логарифмическое тождество . При этом выписывание равенства происходит синхронно с повторным чтением определения теперь уже в обратном, по сравнению с учебником, порядке. Теперь можно записать корень уравнения 2х = 3: х = loga3 и предложить школьникам серию самостоятельных работ.

Логарифмическая функция


Выразим x из равенства y = logax, получим x = ay. Последнее равенство задает функцию x = ay, график которой симметричен графику показательной функции y = ax относительно прямой y = x.


Показательная функция x = ay является монотонной, и, значит, разные значения y соответствуют разным значениям x, но это говорит о том, что y = logax, в свою очередь, является функцией x.


Показательная функция y = ax и логарифмическая функция y = logax являются взаимно обратными. Сравнивая их графики, можно отметить некоторые основные свойства логарифмической функции.


Свойства функции y = logax, a > 0, a ≠ 11:

  1. Функция y = logax определена и непрерывна на множестве положительных чисел.
  2. Область значений функции y = logax — множество действительных чисел.
  3. При 0 < a < 1 функция y = logax является убывающей; при a > 1 функция y = logax является возрастающей.
  4. График функции y = logax проходит через точку (1; 0).
  5. Ось ординат — вертикальная асимптота графика функции y = loga.


Математика: алгебра и начала математического анализа, геометрия. Алгебра и начала математического анализа. Углубленный уровень. 10 класс. Учебник


Учебник входит в учебно-методический комплекс по математике для 10–11 классов, изучающих предмет на углубленном уровне. Теоретический материал в нем разделен на обязательный и дополнительный. Каждая глава завершается домашней контрольной работой, а каждый пункт главы — контрольными вопросами и заданиями. Учебник соответствует Федеральному государственному образовательному стандарту среднего (полного) общего образования, имеет гриф «Рекомендовано» и включен в Федеральный перечень учебников.

Купить

Решение логарифмических уравнений и неравенств на основе свойств логарифмической функции


Освобождаясь от внешнего логарифма, имеющего основание 3, мы ссылаемся на возрастание соответствующей логарифмической функции, то есть на то, что большему значению логарифма соответствует большее значение выражения, стоящего под его знаком. Однако следует иметь в виду, что если функцию y = log3 log0,5(2x + 1) считать логарифмической, то ее аргумент не переменная x, а все выражение log0,5(2x + 1). Если же все-таки рассматривать x как аргумент функции y = log3 log0,5(2x + 1), то эта функция окажется убывающей, так как при увеличении значения x увеличивается значение выражения 2x + 1, уменьшается значение выражения log0,5(2x + 1) и, соответственно, уменьшается значение самой функции.

Свойства логарифмов


Связь двух форм записи соотношения между числами a, b и x (речь о ax = b и x = logab) позволяет получить свойства логарифмов, основываясь на известных свойствах степеней.


Рассмотрим, например, произведение степеней с одинаковым основанием: axay. Пусть x = b и a y = c. Перейдем к логарифмической форме: x = logab и y = logac, тогда bc = a logab × a logac = a logab + logac. От показательной формы равенства bc = a logab + logac перейдем к логарифмической форме:


loga(bc) = logab + logac


Заметим, что в левой части формулы числа a и b могут быть отрицательными. Тогда формула будет выглядеть так:


loga(bc) = loga|b| + loga|c|


Аналогично можно получить еще два свойства для логарифмов частного и степени.

  • логарифм произведения loga (bc) = loga |b| + loga |c|
  • логарифм частного
  • логарифм степени logabp = p loga|b|


Последнее свойство дает возможность вывести важную формулу, с помощью которой можно выразить логарифм с одним основанием через логарифм с другим основанием.


Пусть logab = x. Перейдем к показательной форме ax = b. Прологарифмируем это равенство по основанию c, т.е. найдем логарифмы с основанием c обеих частей этого равенства: logcax = logcb. Применяя к левой части свойство логарифма степени, получим x logca = logcb или , откуда .


Формула перехода от одного основания логарифма к другому


Полезно запомнить частный случай формулы перехода, когда одно из оснований является степенью другого:


Рассмотренные свойства и формула перехода «работают», конечно, только когда все входящие в них выражения имеют смысл.


Что ещё почитать?

Логарифмы на ЕГЭ


Логарифмы встречаются на ЕГЭ: как во второй части (обычно, это задание 15), так и, реже, в первой части. Задания из аттестации — одно из средств мотивации детей на уроках. Зная, что упражнение на доске аналогично заданию ЕГЭ, ученик будет внимательнее следить за его решением.


Разберем несколько таких заданий.

Из первой части (определение логарифма на ЕГЭ профильного уровня)


Решите уравнение log3(x+1)2 + log3|x+1| = 6 . Если корней несколько, укажите наименьший из них.


Решение. Решаем квадратное относительно log3|x+1| уравнение. Его корни 2 и −3.


log3|x+1| = 2, |x+1| = 9, x = −10 — это наименьший из корней.


Ответ: −10.

Из второй части (логарифмическое неравенство на ЕГЭ профильного уровня)


Решите неравенство .


Решение. ОДЗ: x > 0, x ≠ 1. Перейдем к логарифмам по основанию 10:


;


;


Умножим числитель и знаменатель на 2, чтобы уйти от радикала:


;


Нули числителя: 2/3, 3, с учетом положительности x, нуль заменяется на 1.


Ответ:


Алгебра в таблицах. 7-11 классы. Справочное пособие


Пособие содержит таблицы по всем наиболее важным разделам школьного курса арифметики, алгебры, начал анализа. В таблицах кратко изложена теория по каждой теме, приведены основные формулы, графики и примеры решения типовых задач. В конце книги помещен предметный указатель. Пособие будет полезно учащимся 7-11 классов, абитуриентам, студентам, учителям и родителям.

Купить

Из второй части (логарифмическое уравнение с параметром на ЕГЭ профильного уровня)


Найдите все значения a, для которых при любом положительном значении b уравнение имеет хотя бы одно решение, меньше 1/3.


Решение. Найдем ОДЗ:


Стандартно приводим логарифмы к одному основанию


,


.


Получили квадратное уравнение относительно .


Оно должно иметь корень при


Обозначим, что и рассмотрим квадратичную функцию y = t— bt — 2a.


Ветви ее графика направлены вверх, а вершина, поскольку b > 0, расположена в левой координатной полуплоскости. Первая ветвь параболы пересекает ось абсцисс правее t = 0, значит при t = 0 y < 0. Получаем −2a < 0 a > 0.


Ответ: a > 0.


Учебник
«Алгебра и начала математического анализа. Углубленный уровень. 10 класс» схож по структуре с учебником базового уровня, однако предполагает больше часов на изучение сложных задач. Эти и другие издания линейки вы можете апробировать прямо сейчас, воспользовавшись акцией
«5 учебников бесплатно». Методическое пособие представлено в свободном доступе. Приглашаем познакомиться с другими вебинарами экспертов и порекомендовать нам интересующую вас тему для последующих трансляций.


#ADVERTISING_INSERT#


Уравнения с логарифмами. Логарифмические уравнения. Как решать логарифмические уравнения

Как известно, при перемножении выражений со степенями их показатели всегда складываются (a b *a c = a b+c). Этот математический закон был выведен Архимедом, а позже, в VIII веке, математик Вирасен создал таблицу целых показателей. Именно они послужили для дальнейшего открытия логарифмов. Примеры использования этой функции можно встретить практически везде, где требуется упростить громоздкое умножение на простое сложение. Если вы потратите минут 10 на прочтение этой статьи, мы вам объясним, что такое логарифмы и как с ними работать. Простым и доступным языком.

Определение в математике

Логарифмом называется выражение следующего вида: log a b=c, то есть логарифмом любого неотрицательного числа (то есть любого положительного) «b» по его основанию «a» считается степень «c», в которую необходимо возвести основание «a», чтобы в итоге получить значение «b». Разберем логарифм на примерах, допустим, есть выражение log 2 8. Как найти ответ? Очень просто, нужно найти такую степень, чтобы из 2 в искомой степени получить 8. Проделав в уме некоторые расчеты, получаем число 3! И верно, ведь 2 в степени 3 дает в ответе число 8.

Разновидности логарифмов

Для многих учеников и студентов эта тема кажется сложной и непонятной, однако на самом деле логарифмы не так страшны, главное — понять общий их смысл и запомнить их свойста и некоторые правила. Существует три отдельных вида логарифмических выражений:

  1. Натуральный логарифм ln a, где основанием является число Эйлера (e = 2,7).
  2. Десятичный a, где основанием служит число 10.
  3. Логарифм любого числа b по основанию a>1.

Каждый из них решается стандартным способом, включающим в себя упрощение, сокращение и последующее приведение к одному логарифму с помощью логарифмических теорем. Для получения верных значений логарифмов следует запомнить их свойства и очередность действий при их решениях.

Правила и некоторые ограничения

В математике существует несколько правил-ограничений, которые принимаются как аксиома, то есть не подлежат обсуждению и являются истиной. Например, нельзя числа делить на ноль, а еще невозможно извлечь корень четной степени из отрицательных чисел. Логарифмы также имеют свои правила, следуя которым можно с легкостью научиться работать даже с длинными и емкими логарифмическими выражениями:

  • основание «a» всегда должно быть больше нуля, и при этом не быть равным 1, иначе выражение потеряет свой смысл, ведь «1» и «0» в любой степени всегда равны своим значениям;
  • если а > 0, то и а b >0, получается, что и «с» должно быть больше нуля.

Как решать логарифмы?

К примеру, дано задание найти ответ уравнения 10 х = 100. Это очень легко, нужно подобрать такую степень, возведя в которую число десять, мы получим 100. Это, конечно же, 10 2 =100.

А теперь давайте представим данное выражение в виде логарифмического. Получим log 10 100 = 2. При решении логарифмов все действия практически сходятся к тому, чтобы найти ту степень, в которую необходимо ввести основание логарифма, чтобы получить заданное число.

Для безошибочного определения значенияя неизвестной степени необходимо научиться работать с таблицей степеней. Выглядит она следующим образом:

Как видите, некоторые показатели степени можно угадать интуитивно, если имеется технический склад ума и знание таблицы умножения. Однако для больших значений потребуется таблица степеней. Ею могут пользоваться даже те, кто совсем ничего не смыслит в сложных математических темах. В левом столбце указаны числа (основание a), верхний ряд чисел — это значение степени c, в которую возводится число a. На пересечении в ячейках определены значения чисел, являющиеся ответом (a c =b). Возьмем, к примеру, самую первую ячейку с числом 10 и возведем ее в квадрат, получим значение 100, которое указано на пересечении двух наших ячеек. Все так просто и легко, что поймет даже самый настоящий гуманитарий!

Уравнения и неравенства

Получается, что при определенных условиях показатель степени — это и есть логарифм. Следовательно, любые математические численные выражения можно записать в виде логарифмического равенства. Например, 3 4 =81 можно записать в виде логарифма числа 81 по основанию 3, равному четырем (log 3 81 = 4). Для отрицательных степеней правила такие же: 2 -5 = 1/32 запишем в виде логарифма, получим log 2 (1/32) = -5. Одной из самых увлекательных разделов математики является тема «логарифмы». Примеры и решения уравнений мы рассмотрим чуть ниже, сразу же после изучения их свойств. А сейчас давайте разберем, как выглядят неравенства и как их отличить от уравнений.

Дано выражение следующего вида: log 2 (x-1) > 3 — оно является логарифмическим неравенством, так как неизвестное значение «х» находится под знаком логарифма. А также в выражении сравниваются две величины: логарифм искомого числа по основанию два больше, чем число три.

Самое главное отличие между логарифмическими уравнениями и неравенствами заключается в том, что уравнения с логарифмами (пример — логарифм 2 x = √9) подразумевают в ответе одно или несколько определенных числовых значений, тогда как при решении неравенства определяются как область допустимых значений, так и точки разрыва этой функции. Как следствие, в ответе получается не простое множество отдельных чисел как в ответе уравнения, а а непрерывный ряд или набор чисел.

Основные теоремы о логарифмах

При решении примитивных заданий по нахождению значений логарифма, его свойства можно и не знать. Однако когда речь заходит о логарифмических уравнениях или неравенствах, в первую очередь, необходимо четко понимать и применять на практике все основные свойства логарифмов. С примерами уравнений мы познакомимся позже, давайте сначала разберем каждое свойство более подробно.

  1. Основное тождество выглядит так: а logaB =B. Оно применяется только при условии, когда а больше 0, не равно единице и B больше нуля.
  2. Логарифм произведения можно представить в следующей формуле: log d (s 1 *s 2) = log d s 1 + log d s 2. При этом обязательным условием является: d, s 1 и s 2 > 0; а≠1. Можно привести доказательство для этой формулы логарифмов, с примерами и решением. Пусть log a s 1 = f 1 и log a s 2 = f 2 , тогда a f1 = s 1 , a f2 = s 2. Получаем, что s 1 *s 2 = a f1 *a f2 = a f1+f2 (свойства степеней), а далее по определению: log a (s 1 *s 2)= f 1 + f 2 = log a s1 + log a s 2, что и требовалось доказать.
  3. Логарифм частного выглядит так: log a (s 1/ s 2) = log a s 1 — log a s 2.
  4. Теорема в виде формулы приобретает следующий вид: log a q b n = n/q log a b.

Называется эта формула «свойством степени логарифма». Она напоминает собой свойства обычных степеней, и неудивительно, ведь вся математика держится на закономерных постулатах. Давайте посмотрим на доказательство.

Пусть log a b = t, получается a t =b. Если возвести обе части в степень m: a tn = b n ;

но так как a tn = (a q) nt/q = b n , следовательно log a q b n = (n*t)/t, тогда log a q b n = n/q log a b. Теорема доказана.

Примеры задач и неравенств

Самые распространенные типы задач на тему логарифмов — примеры уравнений и неравенств. Они встречаются практически во всех задачниках, а также входят в обязательную часть экзаменов по математике. Для поступления в университет или сдачи вступительных испытаний по математике необходимо знать, как правильно решать подобные задания.

К сожалению, единого плана или схемы по решению и определению неизвестного значения логарифма не существует, однако к каждому математическому неравенству или логарифмическому уравнению можно применить определенные правила. Прежде всего следует выяснить, можно ли упростить выражение или привести к общему виду. Упрощать длинные логарифмические выражения можно, если правильно использовать их свойства. Давайте скорее с ними познакомимся.

При решении же логарифмических уравнений, следует определить, какой перед нами вид логарифма: пример выражения может содержать натуральный логарифм или же десятичный.

Вот примеры ln100, ln1026. Их решение сводится к тому, что нужно определить ту степень, в которой основание 10 будет равно 100 и 1026 соответственно. Для решений же натуральных логарифмов нужно применить логарифмические тождества или же их свойства. Давайте на примерах рассмотрим решение логарифмических задач разного типа.

Как использовать формулы логарифмов: с примерами и решениями

Итак, рассмотрим примеры использования основных теорем о логарифмах.

  1. Свойство логарифма произведения можно применять в заданиях, где необходимо разложить большое значение числа b на более простые сомножители. Например, log 2 4 + log 2 128 = log 2 (4*128) = log 2 512. Ответ равен 9.
  2. log 4 8 = log 2 2 2 3 = 3/2 log 2 2 = 1,5 — как видите, применяя четвертое свойство степени логарифма, удалось решить на первый взгляд сложное и нерешаемое выражение. Необходимо всего лишь разложить основание на множители и затем вынести значения степени из знака логарифма.

Задания из ЕГЭ

Логарифмы часто встречаются на вступительных экзаменах, особенно много логарифмических задач в ЕГЭ (государственный экзамен для всех выпускников школ). Обычно эти задания присутствуют не только в части А (самая легкая тестовая часть экзамена), но и в части С (самые сложные и объемные задания). Экзамен подразумевает точное и идеальное знание темы «Натуральные логарифмы».

Примеры и решения задач взяты из официальных вариантов ЕГЭ. Давайте посмотрим, как решаются такие задания.

Дано log 2 (2x-1) = 4. Решение:
перепишем выражение, немного его упростив log 2 (2x-1) = 2 2 , по определению логарифма получим, что 2x-1 = 2 4 , следовательно 2x = 17; x = 8,5.

  • Все логарифмы лучше всего приводить к одному основанию, чтобы решение не было громоздким и запутанным.
  • Все выражение, стоящие под знаком логарифма, указываются как положительные, поэтому при вынесении множителем показателя степени выражения, который стоит под знаком логарифма и в качестве его основания, остающееся под логарифмом выражение должно быть положительно.

Заключительные видео из длинной серии уроков про решение логарифмических уравнений. В этот раз мы будем работать в первую очередь с ОДЗ логарифма — именно из-за неправильного учета (или вообще игнорирования) области определения возникает большинство ошибок при решении подобных задач.

В этом коротком видеоуроке мы разберем применение формул сложения и вычитания логарифмов, а также разберемся с дробно-рациональными уравнениями, с которыми у многих учеников также возникают проблемы.

О чем пойдет речь? Главная формула, с которой я хотел бы разобраться, выглядит так:

log a
(f
g
) = log a
f
+ log a
g

Это стандартный переход от произведения к сумме логарифмов и обратно. Вы наверняка знаете эту формулу с самого начала изучения логарифмов. Однако тут есть одна заминка.

До тех пор, пока в виде переменных a
, f
и g
выступают обычные числа, никаких проблем не возникает. Данная формула работает прекрасно.

Однако, как только вместоf
и g
появляются функции, возникает проблема расширения или сужения области определения в зависимости от того, в какую сторону преобразовывать. Судите сами: в логарифме, записанном слева, область определения следующая:

fg
> 0

А вот в сумме, записанной справа, область определения уже несколько иная:

f
> 0

g
> 0

Данный набор требований является более жестким, чем исходный. В первом случае нас устроит вариант f
0 выполняется).

Итак, при переходе от левой конструкции к правой возникает сужение области определения. Если же сначала у нас была сумма, а мы переписываем ее в виде произведения, то происходит расширение области определения.

Другими словами, в первом случае мы могли потерять корни, а во втором — получить лишние. Это необходимо учитывать при решении реальных логарифмических уравнений.

Итак, первая задача:

[Подпись к рисунку]

Слева мы видим сумму логарифмов по одному и тому же основанию. Следовательно, эти логарифмы можно сложить:

[Подпись к рисунку]

Как видите, справа мы заменил ноль по формуле:

a
= log b
b
a

Давайте еще немного преобразуем наше уравнение:

log 4 (x
− 5) 2 = log 4 1

Перед нами каноническая форма логарифмического уравнения, мы можем зачеркнуть знак log и приравнять аргументы:

(x
− 5) 2 = 1

|x
− 5| = 1

Обратите внимание: откуда взялся модуль? Напомню, что корень из точного квадрата равен именно модулю:

[Подпись к рисунку]

Затем решаем классическое уравнение с модулем:

|f
| = g
(g
> 0) ⇒f
= ±g

x
− 5 = ±1 ⇒x
1 = 5 − 1 = 4; x
2 = 5 + 1 = 6

Вот два кандидат на ответ. Являются ли они решением исходного логарифмического уравнения? Нет, ни в коем случае!

Оставить все просто так и записать ответ мы не имеем права. Взгляните на тот шаг, когда мы заменяем сумму логарифмов одним логарифмом от произведения аргументов. Проблема в том, что в исходных выражениях у нас стоят функции. Следовательно, следует потребовать:

х(х − 5) > 0; (х − 5)/х > 0.

Когда же мы преобразовали произведение, получив точный квадрат, требования изменились:

(x
− 5) 2 > 0

Когда это требование выполняется? Да практически всегда! За исключением того случая, когда х − 5 = 0. Т.е. неравенство сведется к одной выколотой точке:

х − 5 ≠ 0 ⇒ х ≠ 5

Как видим, произошло расширение области определения, о чем мы и говорили в самом начале урока. Следовательно, могут возникнуть и лишние корни.

Как же не допустить возникновения этих лишних корней? Очень просто: смотрим на наши полученные корни и сравниваем их с областью определения исходного уравнения. Давайте посчитаем:

х (х − 5) > 0

Решать будем с помощью метода интервалов:

х (х − 5) = 0 ⇒ х = 0; х = 5

Отмечаем полученные числа на прямой. Все точки выколотые, потому что неравенство строгое. Берем любое число, больше 5 и подставляем:

[Подпись к рисунку]

На интересуют промежутки (−∞; 0) ∪ (5; ∞). Если мы отметим наши корни на отрезке, то увидим, что х = 4 нас не устраивает, потому что этот корень лежит за пределами области определения исходного логарифмического уравнения.

Возвращаемся к совокупности, вычеркиваем корень х = 4 и записываем ответ: х = 6. Это уже окончательный ответ к исходному логарифмическому уравнению. Все, задача решена.

Переходим ко второму логарифмическому уравнению:

[Подпись к рисунку]

Решаем его. Заметим, что первое слагаемое представляет собой дробь, а второе — ту же самую дробь, но перевернутую. Не пугайтесь выражения lgx
— это просто десятичный логарифм, мы можем записать:

lgx
= log 10 x

Поскольку перед нами две перевернутые дроби, предлагаю ввести новую переменную:

[Подпись к рисунку]

Следовательно, наше уравнение может быть переписано следующим образом:

t
+ 1/t
= 2;

t
+ 1/t
− 2 = 0;

(t
2 − 2t
+ 1)/t
= 0;

(t
− 1) 2 /t
= 0.

Как видим, в числителе дроби стоит точный квадрат. Дробь равна нулю, когда ее числитель равен нулю, а знаменатель отличен от нуля:

(t
− 1) 2 = 0; t
≠ 0

Решаем первое уравнение:

t
− 1 = 0;

t
= 1.

Это значение удовлетворяет второму требованию. Следовательно, можно утверждать, что мы полностью решили наше уравнение, но только относительно переменной t
. А теперь вспоминаем, что такое t
:

[Подпись к рисунку]

Получили пропорцию:

lgx
= 2 lgx
+ 1

2 lgx
− lgx
= −1

lgx
= −1

Приводим это уравнение к канонической форме:

lgx
= lg 10 −1

x
= 10 −1 = 0,1

В итоге мы получили единственный корень, который, по идее, является решением исходного уравнения. Однако давайте все-таки подстрахуемся и выпишем область определения исходного уравнения:

[Подпись к рисунку]

Следовательно, наш корень удовлетворяет всем требованиям. Мы нашли решение исходного логарифмического уравнения. Ответ: x
= 0,1. Задача решена.

Ключевой момент в сегодняшнем уроке один: при использовании формулы перехода от произведения к сумме и обратно обязательно учитывайте, что область определения может сужаться либо расширяться в зависимости от того, в какую сторону выполняется переход.

Как понять, что происходит: сужение или расширение? Очень просто. Если раньше функции были вместе, а теперь стали по отдельности, то произошло сужение области определения (потому что требований стало больше). Если же сначала функции стояли отдельно, а теперь — вместе, то происходит расширение области определения (на произведение накладывается меньше требований, чем на отдельные множители).

С учетом данного замечания хотел бы отметить, что второе логарифмическое уравнение вообще не требует данных преобразований, т. е. мы нигде не складываем и не перемножаем аргументы. Однако здесь я хотел бы обратить ваше внимание на другой замечательный прием, который позволяет существенно упростить решение. Речь идет о замене переменной.

Однако помните, что никакие замены не освобождает нас от области определения. Именно поэтому после того были найдены все корни, мы не поленились и вернулись к исходному уравнению, чтобы найти его ОДЗ.

Часто при замене переменной возникает обидная ошибка, когда ученики находят значение t
и думают, что на этом решение закончено. Нет, ни в коем случае!

Когда вы нашли значение t
, необходимо вернуться к исходному уравнению и посмотреть, что именно мы обозначали этой буквой. В результате нам предстоит решить еще одно уравнение, которое, впрочем, будет значительно проще исходного.

Именно в этом состоит смысл введения новой переменной. Мы разбиваем исходное уравнение на два промежуточных, каждое из которых решается существенно проще.

Как решать «вложенные» логарифмические уравнения

Сегодня мы продолжаем изучать логарифмические уравнения и разберем конструкции, когда один логарифм стоит под знаком другого логарифма. Оба уравнения мы будем решать с помощью канонической формы.

Сегодня мы продолжаем изучать логарифмические уравнения и разберем конструкции, когда один логарифм стоит под знаком другого. Оба уравнения мы будем решать с помощью канонической формы. Напомню, если у нас есть простейшее логарифмическое уравнение вида log a
f
(x
) = b
, то для решения такого уравнения мы выполняем следующие шаги. В первую очередь, нам нужно заменить число b
:

b
= log a
a
b

Заметьте: a
b
— это аргумент. Точно так же в исходном уравнении аргументом является функция f
(x
). Затем мы переписываем уравнение и получаем вот такую конструкцию:

log a
f
(x
) = log a
a
b

Уже затем мы можем выполнить третий шаг — избавится от знака логарифма и просто записать:

f
(x
) = a
b

В результате мы получим новое уравнение. При этом никаких ограничений на функцию f
(x
) не накладывается. Например, на ее месте также может стоять логарифмическая функция. И тогда мы вновь получим логарифмическое уравнение, которое снова сведем к простейшему и решим через каноническую форму.

Впрочем, хватит лирики. Давайте решим настоящую задачу. Итак, задача № 1:

log 2 (1 + 3 log 2 x
) = 2

Как видим, перед нами простейшее логарифмическое уравнение. В роли f
(x
) выступает конструкция 1 + 3 log 2 x
, а в роли числа b
выступает число 2 (в роли a
также выступает двойка). Давайте перепишем эту двойку следующим образом:

Важно понимать, что первые две двойки пришли к нам из основания логарифма, т. е. если бы в исходном уравнении стояла 5, то мы бы получили, что 2 = log 5 5 2 . В общем, основание зависит исключительно от логарифма, который изначально дан в задаче. И в нашем случае это число 2.

Итак, переписываем наше логарифмическое уравнение с учетом того, что двойка, которая стоит справа, на самом деле тоже является логарифмом. Получим:

log 2 (1 + 3 log 2 x
) = log 2 4

Переходим к последнему шагу нашей схемы — избавляемся от канонической формы. Можно сказать, просто зачеркиваем знаки log. Однако с точки зрения математики «зачеркнуть log» невозможно — правильнее сказать, что мы просто просто приравниваем аргументы:

1 + 3 log 2 x
= 4

Отсюда легко находится 3 log 2 x
:

3 log 2 x
= 3

log 2 x
= 1

Мы вновь получили простейшее логарифмическое уравнение, давайте снова приведем его к канонической форме. Для этого нам необходимо провести следующие изменения:

1 = log 2 2 1 = log 2 2

Почему в основании именно двойка? Потому что в нашем каноническом уравнении слева стоит логарифм именно по основанию 2. Переписываем задачу с учетом этого факта:

log 2 x
= log 2 2

Снова избавляемся от знака логарифма, т. е. просто приравниваем аргументы. Мы вправе это сделать, потому что основания одинаковые, и больше никаких дополнительных действий ни справа, ни слева не выполнялось:

Вот и все! Задача решена. Мы нашли решение логарифмического уравнения.

Обратите внимание! Хотя переменная х и стоит в аргументе (т. е. возникают требования к области определения), мы никаких дополнительных требований предъявлять не будем.

Как я уже говорил выше, данная проверка является избыточной, если переменная встречается лишь в одном аргументе лишь одного логарифма. В нашем случае х действительно стоит лишь в аргументе и лишь под одним знаком log. Следовательно, никаких дополнительных проверок выполнять не требуется.

Тем не менее, если вы не доверяете данному методу, то легко можете убедиться, что х = 2 действительно является корнем. Достаточно подставить это число в исходное уравнение.

Давайте перейдем ко второму уравнению, оно чуть интересней:

log 2 (log 1/2 (2x
− 1) + log 2 4) = 1

Если обозначить выражение внутри большого логарифма функцией f
(x
), получим простейшее логарифмическое уравнение, с которого мы начинали сегодняшний видеоурок. Следовательно, можно применить каноническую форму, для чего придется представить единицу в виде log 2 2 1 = log 2 2.

Переписываем наше большое уравнение:

log 2 (log 1/2 (2x
− 1) + log 2 4) = log 2 2

Изваляемся от знака логарифма, приравнивая аргументы. Мы вправе это сделать, потому что и слева, и справа основания одинаковые. Кроме того, заметим, что log 2 4 = 2:

log 1/2 (2x
− 1) + 2 = 2

log 1/2 (2x
− 1) = 0

Перед нами снова простейшее логарифмическое уравнение вида log a
f
(x
) = b
. Переходим к канонической форме, т. е. представляем ноль в виде log 1/2 (1/2)0 = log 1/2 1.

Переписываем наше уравнение и избавляемся от знака log, приравнивая аргументы:

log 1/2 (2x
− 1) = log 1/2 1

2x
− 1 = 1

Опять же мы сразу получили ответ. Никаких дополнительных проверок не требуется, потому что в исходном уравнении лишь один логарифм содержит функцию в аргументе.

Следовательно, никаких дополнительных проверок выполнять не требуется. Мы можем смело утверждать, что х = 1 является единственным корнем данного уравнения.

А вот если бы во втором логарифме вместо четверки стояла бы какая-то функция от х (либо 2х стояло бы не в аргументе, а в основании) — вот тогда потребовалось бы проверять область определения. Иначе велик шанс нарваться на лишние корни.

Откуда возникают такие лишние корни? Этот момент нужно очень четко понимать. Взгляните на исходные уравнения: везде функция х стоит под знаком логарифма. Следовательно, поскольку мы записали log 2 x
, то автоматически выставляем требование х > 0. Иначе данная запись просто не имеет смысла.

Однако по мере решения логарифмического уравнения мы избавляемся от всех знаков log и получаем простенькие конструкции. Здесь уже никаких ограничений не выставляется, потому что линейная функция определена при любом значении х.

Именно эта проблема, когда итоговая функция определена везде и всегда, а исходная — отнюдь не везде и не всегда, и является причиной, по которой в решении логарифмических уравнениях очень часто возникают лишние корни.

Но повторю еще раз: такое происходить лишь в ситуации, когда функция стоит либо в нескольких логарифмах, либо в основании одного из них. В тех задачах, которые мы рассматриваем сегодня, проблем с расширением области определения в принципе не существует.

Случаи разного основания

Этот урок посвящен уже более сложным конструкциям. Логарифмы в сегодняшних уравнениях уже не будут решаться «напролом» — сначала потребуется выполнить некоторые преобразования.

Начинаем решение логарифмических уравнений с совершенно разными основаниями, которые не являются точными степенями друг друга. Пусть вас не пугают подобные задачи — решаются они ничуть не сложнее, чем самые простые конструкции, которые мы разбирали выше.

Но прежде, чем переходить непосредственно к задачам, напомню о формуле решения простейших логарифмических уравнений с помощью канонической формы. Рассмотрим задачу вот такого вида:

log a
f
(x
) = b

Важно, что функция f
(x
) является именно функцией, а в роли чисел а и b
должны выступать именно числа (без всяких переменных x
). Разумеется, буквально через минуту мы рассмотрим и такие случаи, когда вместо переменных а и b
стоят функции, но сейчас не об этом.

Как мы помним, число b
нужно заменить логарифмом по тому же самому основанию а, которое стоит слева. Это делается очень просто:

b
= log a
a
b

Разумеется, под словом «любое число b
» и «любое число а» подразумеваются такие значения, которые удовлетворяют области определения. В частности, в данном уравнении речь идет лишь основание a
> 0 и a
≠ 1.

Однако данное требование выполняется автоматически, потому что в исходной задаче уже присутствует логарифм по основанию а — оно заведомо будет больше 0 и не равно 1. Поэтому продолжаем решение логарифмического уравнения:

log a
f
(x
) = log a
a
b

Подобная запись называется канонической формой. Ее удобство состоит в том, что мы сразу можем избавиться от знака log, приравняв аргументы:

f
(x
) = a
b

Именно этот прием мы сейчас будем использовать для решения логарифмических уравнений с переменным основанием. Итак, поехали!

log 2 (x
2 + 4x
+ 11) = log 0,5 0,125

Что дальше? Кто-то сейчас скажет, что нужно вычислить правый логарифм, либо свести их к одному основанию, либо что-то еще. И действительно, сейчас нужно привести оба основания к одному виду — либо 2, либо 0,5. Но давайте раз и навсегда усвоим следующее правило:

Если в логарифмическом уравнении присутствуют десятичные дроби, обязательно переведите эти дроби из десятичной записи в обычную. Такое преобразование может существенно упростить решение.

Подобный переход нужно выполнять сразу, еще до выполнения каких-либо действий и преобразований. Давайте посмотрим:

log 2 (x
2 + 4x
+ 11) = log 1 /2 1/8

Что нам дает такая запись? Мы можем 1/2 и 1/8 представить как степень с отрицательным показателем:

[Подпись к рисунку]

Перед нами каноническая форма. Приравниваем аргументы и получаем классическое квадратное уравнение:

x
2 + 4x
+ 11 = 8

x
2 + 4x
+ 3 = 0

Перед нами приведенное квадратное уравнение, которое легко решается с помощью формул Виета. Подобные выкладки в старших классах вы должны видеть буквально устно:

(х + 3)(х + 1) = 0

x
1 = −3

x
2 = −1

Вот и все! Исходное логарифмическое уравнение решено. Мы получили два корня.

Напомню, что определять область определения в данном случае не требуется, поскольку функция с переменной х присутствует лишь в одном аргументе. Поэтому область определения выполняется автоматически.

Итак, первое уравнение решено. Переходим ко второму:

log 0,5 (5x
2 + 9x
+ 2) = log 3 1/9

log 1/2 (5x
2 + 9x
+ 2) = log 3 9 −1

А теперь заметим, что аргумент первого логарифма тоже можно записать в виде степени с отрицательным показателем: 1/2 = 2 −1 . Затем можно вынести степени с обеих сторон уравнения и разделить все на −1:

[Подпись к рисунку]

И вот сейчас мы выполнили очень важный шаг в решении логарифмического уравнения. Возможно, кто-то что-то не заметил, поэтому давайте я поясню.

Взгляните на наше уравнение: и слева, и справа стоит знак log, но слева стоит логарифм по основанию 2, а справа стоит логарифм по основанию 3. Тройка не является целой степенью двойки и, наоборот: нельзя записать, что 2 — это 3 в целой степени.

Следовательно, это логарифмы с разными основаниями, которые не сводятся друг к другу простым вынесением степеней. Единственный путь решения таких задач — избавиться от одного из этих логарифмов. В данном случае, поскольку мы пока рассматриваем довольно простые задачи, логарифм справа просто сосчитался, и мы получили простейшее уравнение — именно такое, о котором мы говорили в самом начале сегодняшнего урока.

Давайте представим число 2, которое стоит справа в виде log 2 2 2 = log 2 4. А затем избавимся от знака логарифма, после чего у нас остается просто квадратное уравнение:

log 2 (5x
2 + 9x
+ 2) = log 2 4

5x
2 + 9x
+ 2 = 4

5x
2 + 9x
− 2 = 0

Перед нами обычное квадратное уравнение, однако оно не является приведенным, потому что коэффициент при x
2 отличен от единицы. Следовательно, решать мы его будем с помощью дискриминанта:

D
= 81 − 4 5 (−2) = 81 + 40 = 121

x
1 = (−9 + 11)/10 = 2/10 = 1/5

x
2 = (−9 − 11)/10 = −2

Вот и все! Мы нашли оба корня, а значит, получили решение исходного логарифмического уравнения. Ведь в исходной задачи функция с переменной х присутствует лишь в одном аргументе. Следовательно, никаких дополнительных проверок на область определения не требуется — оба корня, которые мы нашли, заведомо отвечают всем возможным ограничениям.

На этом можно было бы закончить сегодняшний видеоурок, но в заключении я хотел бы сказать еще раз: обязательно переводите все десятичные дроби в обычные при решении логарифмических уравнений. В большинстве случаев это существенно упрощает их решение.

Редко, очень редко попадаются задачи, в которых избавление от десятичных дробей лишь усложняет выкладки. Однако в таких уравнениях, как правило, изначально видно, что избавляться от десятичных дробей не надо.

В большинстве остальных случаев (особенно если вы только начинаете тренироваться в решении логарифмических уравнений) смело избавляйтесь от десятичных дробей и переводите их в обычные. Потому что практика показывает, что таким образом вы значительно упростите последующее решение и выкладки.

Тонкости и хитрости решения

Сегодня мы переходим к более сложным задачам и будем решать логарифмическое уравнение, в основании которого стоит не число, а функция.

И пусть даже эта функция линейна — в схему решения придется внести небольшие изменения, смысл которых сводится к дополнительным требованиям, накладываемым на область определения логарифма.

Сложные задачи

Этот урок будет довольно длинным. В нем мы разберем два довольно серьезных логарифмических уравнения, при решении которых многие ученики допускают ошибки. За свою практику работы репетитором по математике я постоянно сталкивался с двумя видами ошибок:

  1. Возникновение лишних корней из-за расширения области определения логарифмов. Чтобы не допускать такие обидные ошибки, просто внимательно следите за каждым преобразованием;
  2. Потери корней из-за того, что ученик забыл рассмотреть некоторые «тонкие» случаи — именно на таких ситуациях мы сегодня и сосредоточимся.

Это последний урок, посвященный логарифмическим уравнениям. Он будет длинным, мы разберем сложные логарифмические уравнения. Устраивайтесь поудобней, заварите себе чай, и мы начинаем.

Первое уравнение выглядит вполне стандартно:

log x
+ 1 (x
− 0,5) = log x
− 0,5 (x
+ 1)

Сразу заметим, что оба логарифма являются перевернутыми копиями друг друга. Вспоминаем замечательную формулу:

log a
b
= 1/log b
a

Однако у этой формулы есть ряд ограничений, которые возникают в том случае, если вместо чисел а и b
стоят функции от переменной х:

b
> 0

1 ≠ a
> 0

Эти требования накладываются на основание логарифма. С другой стороны, в дроби от нас требуется 1 ≠ a
> 0, поскольку не только переменная a
стоит в аргументе логарифма (следовательно, a
> 0), но и сам логарифм находится в знаменателе дроби. Но log b
1 = 0, а знаменатель должен быть отличным от нуля, поэтому a
≠ 1.

Итак, ограничения на переменную a
сохраняется. Но что происходит с переменной b
? С одной стороны, из основания следует b
> 0, с другой — переменная b
≠ 1, потому что основание логарифма должно быть отлично от 1. Итого из правой части формулы следует, что 1 ≠ b
> 0.

Но вот беда: второе требование (b
≠ 1) отсутствует в первом неравенстве, посвященном левому логарифму. Другими словами, при выполнении данного преобразования мы должны отдельно проверить
, что аргумент b
отличен от единицы!

Вот давайте и проверим. Применим нашу формулу:

[Подпись к рисунку]

1 ≠ х − 0,5 > 0; 1 ≠ х + 1 > 0

Вот мы и получили, что уже из исходного логарифмического уравнения следует, что и а, и b
должны быть больше 0 и не равны 1. Значит, мы спокойно можем переворачивать логарифмическое уравнение:

Предлагаю ввести новую переменную:

log x
+ 1 (x
− 0,5) = t

В этом случае наша конструкция перепишется следующим образом:

(t
2 − 1)/t
= 0

Заметим, что в числителе у нас стоит разность квадратов. Раскрываем разность квадратов по формуле сокращенного умножения:

(t
− 1)(t
+ 1)/t
= 0

Дробь равна нулю, когда ее числитель равен нулю, а знаменатель отличен от нуля. Но в числителе стоит произведение, поэтому приравниваем к нулю каждый множитель:

t
1 = 1;

t
2 = −1;

t
≠ 0.

Как видим, оба значения переменной t
нас устраивают. Однако на этом решение не заканчивается, ведь нам требуется найти не t
, а значение x
. Возвращаемся к логарифму и получаем:

log x
+ 1 (x
− 0,5) = 1;

log x
+ 1 (x
− 0,5) = −1.

Давайте приведем каждое из этих уравнений к канонической форме:

log x
+ 1 (x
− 0,5) = log x
+ 1 (x
+ 1) 1

log x
+ 1 (x
− 0,5) = log x
+ 1 (x
+ 1) −1

Избавляемся от знака логарифма в первом случае и приравниваем аргументы:

х − 0,5 = х + 1;

х − х = 1 + 0,5;

Такое уравнение не имеет корней, следовательно, первое логарифмическое уравнение также не имеет корней. А вот со вторым уравнением все намного интересней:

(х − 0,5)/1 = 1/(х + 1)

Решаем пропорцию — получим:

(х − 0,5)(х + 1) = 1

Напоминаю, что при решении логарифмических уравнений гораздо удобней приводить все десятичные дроби обычные, поэтому давайте перепишем наше уравнение следующим образом:

(х − 1/2)(х + 1) = 1;

x
2 + x
− 1/2x
− 1/2 − 1 = 0;

x
2 + 1/2x
− 3/2 = 0.

Перед нами приведенное квадратное уравнение, оно легко решается по формулам Виета:

(х + 3/2) (х − 1) = 0;

x
1 = −1,5;

x
2 = 1.

Получили два корня — они являются кандидатами на решение исходного логарифмического уравнения. Для того чтобы понять, какие корни действительно пойдут в ответ, давайте вернемся к исходной задаче. Сейчас мы проверим каждый из наших корней на предмет соответствия области определения:

1,5 ≠ х > 0,5; 0 ≠ х > −1.

Эти требования равносильны двойному неравенству:

1 ≠ х > 0,5

Отсюда сразу видим, что корень х = −1,5 нас не устраивает, а вот х = 1 вполне устраивает. Поэтому х = 1 — окончательное решение логарифмического уравнения.

Переходим ко второй задаче:

log x
25 + log 125 x
5 = log 25 x
625

На первый взгляд может показаться, что у всех логарифмов разные основания и разные аргументы. Что делать с такими конструкциями? В первую очередь заметим, что числа 25, 5 и 625 — это степени 5:

25 = 5 2 ; 625 = 5 4

А теперь воспользуемся замечательным свойством логарифма. Дело в том, что можно выносить степени из аргумента в виде множителей:

log a
b
n
= n
∙ log a
b

На данное преобразование также накладываются ограничения в том случае, когда на месте b
стоит функция. Но у нас b
— это просто число, и никаких дополнительных ограничений не возникает. Перепишем наше уравнение:

2 ∙ log x
5 + log 125 x
5 = 4 ∙ log 25 x
5

Получили уравнение с тремя слагаемыми, содержащими знак log. Причем аргументы всех трех логарифмов равны.

Самое время перевернуть логарифмы, чтобы привести их к одному основанию — 5. Поскольку в роли переменной b
выступает константа, никаких изменений области определения не возникает. Просто переписываем:

[Подпись к рисунку]

Как и предполагалось, в знаменателе «вылезли» одни и те же логарифмы. Предлагаю выполнить замену переменной:

log 5 x
= t

В этом случае наше уравнение будет переписано следующим образом:

Выпишем числитель и раскроем скобки:

2 (t
+ 3) (t
+ 2) + t
(t
+ 2) − 4t
(t
+ 3) = 2 (t
2 + 5t
+ 6) + t
2 + 2t
− 4t
2 − 12t
= 2t
2 + 10t
+ 12 + t
2 + 2t
− 4t
2 − 12t
= −t
2 + 12

Возвращаемся к нашей дроби. Числитель должен быть равен нулю:

[Подпись к рисунку]

А знаменатель — отличен от нуля:

t
≠ 0; t
≠ −3; t
≠ −2

Последние требования выполняются автоматически, поскольку все они «завязаны» на целые числа, а все ответы — иррациональные.

Итак, дробно-рациональное уравнение решено, значения переменной t
найдены. Возвращаемся к решению логарифмического уравнения и вспоминаем, что такое t
:

[Подпись к рисунку]

Приводим это уравнение к канонической форме, получим число с иррациональной степенью. Пусть это вас не смущает — даже такие аргументы можно приравнять:

[Подпись к рисунку]

У нас получилось два корня. Точнее, два кандидата в ответы — проверим их на соответствие области определения. Поскольку в основании логарифма стоит переменная х, потребуем следующее:

1 ≠ х > 0;

С тем же успехом утверждаем, что х ≠ 1/125, иначе основание второго логарифма обратится в единицу. Наконец, х ≠ 1/25 для третьего логарифма.

Итого мы получили четыре ограничения:

1 ≠ х > 0; х ≠ 1/125; х ≠ 1/25

А теперь вопрос: удовлетворяют ли наши корни указанным требованиям? Конечно удовлетворяют! Потому что 5 в любой степени будет больше нуля, и требование х > 0 выполняется автоматически.

С другой стороны, 1 = 5 0 , 1/25 = 5 −2 , 1/125 = 5 −3 , а это значит, что данные ограничения для наших корней (у которых, напомню, в показателе стоит иррациональное число) также выполнены, и оба ответа являются решениями задачи.

Итак, мы получили окончательный ответ. Ключевых моментов в данной задаче два:

  1. Будьте внимательны при перевороте логарифма, когда аргумент и основание меняются местами. Подобные преобразования накладывают лишние ограничения на область определения.
  2. Не бойтесь преобразовывать логарифмы: их можно не только переворачивать, но и раскрывать по формуле суммы и вообще менять по любым формулам, которые вы изучали при решении логарифмических выражений. Однако при этом всегда помните: некоторые преобразования расширяют область определения, а некоторые — сужают.

Подготовка к итоговому тестированию по математике включает в себя важный раздел — «Логарифмы». Задания из этой темы обязательно содержатся в ЕГЭ. Опыт прошлых лет показывает, что логарифмические уравнения вызвали затруднения у многих школьников. Поэтому понимать, как найти правильный ответ, и оперативно справляться с ними должны учащиеся с различным уровнем подготовки.

Сдайте аттестационное испытание успешно с помощью образовательного портала «Школково»!

При подготовке к единому государственному экзамену выпускникам старших классов требуется достоверный источник, предоставляющий максимально полную и точную информацию для успешного решения тестовых задач. Однако учебник не всегда оказывается под рукой, а поиск необходимых правил и формул в Интернете зачастую требует времени.

Образовательный портал «Школково» позволяет заниматься подготовкой к ЕГЭ в любом месте в любое время. На нашем сайте предлагается наиболее удобный подход к повторению и усвоению большого количества информации по логарифмам, а также по с одним и несколькими неизвестными. Начните с легких уравнений. Если вы справились с ними без труда, переходите к более сложным. Если у вас возникли проблемы с решением определенного неравенства, вы можете добавить его в «Избранное», чтобы вернуться к нему позже.

Найти необходимые формулы для выполнения задачи, повторить частные случаи и способы вычисления корня стандартного логарифмического уравнения вы можете, заглянув в раздел «Теоретическая справка». Преподаватели «Школково» собрали, систематизировали и изложили все необходимые для успешной сдачи материалы в максимально простой и понятной форме.

Чтобы без затруднений справляться с заданиями любой сложности, на нашем портале вы можете ознакомиться с решением некоторых типовых логарифмических уравнений. Для этого перейдите в раздел «Каталоги». У нас представлено большое количество примеров, в том числе с уравнениями профильного уровня ЕГЭ по математике.

Воспользоваться нашим порталом могут учащиеся из школ по всей России. Для начала занятий просто зарегистрируйтесь в системе и приступайте к решению уравнений. Для закрепления результатов советуем возвращаться на сайт «Школково» ежедневно.

С уравнениями мы все знакомы с начальных классов. Еще там мы учились решать самые простые примеры, и надо признать, что они находят свое применение даже в высшей математике. С уравнениями все просто, в том числи и с квадратными. Если у вас проблемы с этой темой, настоятельно рекомендуем вам повторить ее.

Логарифмы вы, вероятно, тоже уже прошли. Тем не менее, считаем важным рассказать, что это для тех, кто еще не знает. Логарифм приравнивается к степени, в которую нужно возвести основание, чтобы получилось число, стоящее справа от знака логарифма. Приведем пример, исходя из которого, вам все станет ясно.

Если вы возведете 3 в четвертую степень получится 81. Теперь подставьте по аналогии числа, и поймете окончательно, как решаются логарифмы. Теперь осталось лишь совместить два рассмотренных понятия. Изначально ситуация кажется чрезвычайно сложной, но при ближайшем рассмотрении весе становится на свои места. Мы уверены, что после этой короткой статьи у вас не будет проблем в этой части ЕГЭ.

Сегодня выделяют множество способов решения подобных конструкций. Мы расскажем о самых простых, эффективных и наиболее применимых в случае заданий ЕГЭ. Решение логарифмических уравнений должно начинаться с самого простого примера. Простейшие логарифмические уравнения состоят из функции и одной переменной в ней.

Важно учесть, что x находится внутри аргумента. A и b должны быть числами. В таком случае вы можете попросту выразить функцию через число в степени. Выглядит это следующим образом.

Разумеется, решение логарифмического уравнения таким методом приведет вас к верному ответу. Ног проблема подавляющего большинства учеников в этом случае заключается в том, что они не понимают, что и откуда берется. В результате приходится мириться с ошибками и не получать желаемых баллов. Самой обидной ошибкой будет, если вы перепутаете буквы местами. Чтобы решить уравнение этим способом, нужно зазубрить эту стандартную школьную формулу, потому что понять ее сложно.

Чтобы было проще, можно прибегнуть к другому способу – канонической форме. Идея крайне проста. Снова обратите внимание на задачу. Помните, что буква a – число, а не функция или переменная. A не равно одному и больше нуля. На b никаких ограничений не действует. Теперь из всех формул вспоминаем одну. B можно выразить следующим образом.

Из этого следует, что все исходные уравнения с логарифмами можно представить в виде:

Теперь мы можем отбросить логарифмы. Получится простая конструкция, которую мы уже видели ранее.

Удобство данной формулы заключается в том, что ее можно применять в самых разных случаях, а не только для самых простых конструкций.

Не переживайте насчет ООФ!

Многие опытные математики заметят, что мы не уделили внимание области определения. Сводится правило к тому, что F(x) обязательно больше 0. Нет, мы не упустили этот момент. Сейчас мы говорим об еще одном серьезном преимуществе канонической формы.

Лишних корней здесь не возникнет. Если переменная будет встречаться лишь в одном месте, то область определения не является необходимостью. Она выполняется автоматически. Чтобы убедиться в данном суждении, займитесь решением нескольких простых примеров.

Как решать логарифмические уравнения с разными основаниями

Это уже сложные логарифмические уравнения, и подход к их решению должен быть особым. Здесь редко получается ограничиться пресловутой канонической формой. Начнем наш подробный рассказ. Мы имеем следующую конструкцию.

Обратите внимание на дробь. В ней находится логарифм. Если вы увидите такое в задании, стоит вспомнить один интересный прием.

Что это значит? Каждый логарифм можно представить в виде частного двух логарифмов с удобным основанием. И у данной формулы есть частный случай, который применим с этим примером (имеем ввиду, если c=b).

Именно такую дробь мы и видим в нашем примере. Таким образом.

По сути, перевернули дробь и получили более удобное выражение. Запомните этот алгоритм!

Теперь нужно, что логарифмическое уравнение не содержало разных оснований. Представим основание дробью.

В математике есть правило, исходя из которого, можно вынести степень из основания. Получается следующая конструкция.

Казалось бы, что мешает теперь превратить наше выражение в каноническую форму и элементарно решить ее? Не все так просто. Дробей перед логарифмом быть не должно. Исправляем эту ситуацию! Дробь разрешается выносить в качестве степени.

Соответственно.

Если основания одинаковые, мы можем убрать логарифмы и приравнять сами выражения. Так ситуация станет в разы проще, чем была. Останется элементарное уравнение, которое каждый из нас умел решать еще в 8 или даже в 7 классе. Расчеты вы сможете произвести сами.

Мы получили единственно верный корень этого логарифмического уравнения. Примеры решения логарифмического уравнения достаточно просты, не так ли? Теперь и у вас получится самостоятельно разобраться даже с самыми сложными задачами для подготовки и сдачи ЕГЭ.

Что в итоге?

В случае с любыми логарифмическими уравнениями мы исходим из одного очень важного правила. Необходимо действовать так, чтобы привести выражение к максимально простому виду. В таком случае у вас будет больше шансов не просто решить задание правильно, но еще и сделать это максимально простым и логичным путем. Именно так всегда действуют математики.

Настоятельно не рекомендуем вам искать сложных путей, особенно в этом случае. Запомните несколько простых правил, которые позволят преобразовать любое выражение. К примеру, привести два или три логарифма к одному основанию или вывести степень из основания и выиграть на этом.

Также стоит помнить о том, что в решении логарифмических уравнений необходимо постоянно тренироваться. Постепенно вы будете переходить ко все более сложным конструкциям, а это приведет вас к уверенному решению всех вариантов задач на ЕГЭ. Готовьтесь к экзаменам заблаговременно, и удачи вам!

Соблюдение Вашей конфиденциальности важно для нас. По этой причине, мы разработали Политику Конфиденциальности, которая описывает, как мы используем и храним Вашу информацию. Пожалуйста, ознакомьтесь с нашими правилами соблюдения конфиденциальности и сообщите нам, если у вас возникнут какие-либо вопросы.

Сбор и использование персональной информации

Под персональной информацией понимаются данные, которые могут быть использованы для идентификации определенного лица либо связи с ним.

От вас может быть запрошено предоставление вашей персональной информации в любой момент, когда вы связываетесь с нами.

Ниже приведены некоторые примеры типов персональной информации, которую мы можем собирать, и как мы можем использовать такую информацию.

Какую персональную информацию мы собираем:

  • Когда вы оставляете заявку на сайте, мы можем собирать различную информацию, включая ваши имя, номер телефона, адрес электронной почты и т.д.

Как мы используем вашу персональную информацию:

  • Собираемая нами персональная информация позволяет нам связываться с вами и сообщать об уникальных предложениях, акциях и других мероприятиях и ближайших событиях.
  • Время от времени, мы можем использовать вашу персональную информацию для отправки важных уведомлений и сообщений.
  • Мы также можем использовать персональную информацию для внутренних целей, таких как проведения аудита, анализа данных и различных исследований в целях улучшения услуг предоставляемых нами и предоставления Вам рекомендаций относительно наших услуг.
  • Если вы принимаете участие в розыгрыше призов, конкурсе или сходном стимулирующем мероприятии, мы можем использовать предоставляемую вами информацию для управления такими программами.

Раскрытие информации третьим лицам

Мы не раскрываем полученную от Вас информацию третьим лицам.

Исключения:

  • В случае если необходимо — в соответствии с законом, судебным порядком, в судебном разбирательстве, и/или на основании публичных запросов или запросов от государственных органов на территории РФ — раскрыть вашу персональную информацию. Мы также можем раскрывать информацию о вас если мы определим, что такое раскрытие необходимо или уместно в целях безопасности, поддержания правопорядка, или иных общественно важных случаях.
  • В случае реорганизации, слияния или продажи мы можем передать собираемую нами персональную информацию соответствующему третьему лицу – правопреемнику.

Защита персональной информации

Мы предпринимаем меры предосторожности — включая административные, технические и физические — для защиты вашей персональной информации от утраты, кражи, и недобросовестного использования, а также от несанкционированного доступа, раскрытия, изменения и уничтожения.

Соблюдение вашей конфиденциальности на уровне компании

Для того чтобы убедиться, что ваша персональная информация находится в безопасности, мы доводим нормы соблюдения конфиденциальности и безопасности до наших сотрудников, и строго следим за исполнением мер соблюдения конфиденциальности.

{i \ Theta} $ определяется формулой
\ begin {eqnarray} \ label {log2}
\ log z = \ ln r + i \ left (\ Theta + 2n \ pi \ right) \ quad \ quad (n \ in \ mathbb Z).
\ end {eqnarray}

Пример 1: Вычислить $ \ log z $ для $ z = -1- \ sqrt {3} i $.

Решение: Если $ z = -1- \ sqrt {3} i $, то $ r = 2 $ и $ \ Theta = — \ frac {2 \ pi} {3} $. Следовательно
$$ \ log (-1- \ sqrt {3} i) = \ ln 2 + i \ left (- \ frac {2 \ pi} {3} + 2n \ pi \ right) = \ ln 2 +2 \ left (n- \ frac {1} {3} \ right) \ pi i $$
с $ n \ in \ mathbb Z. $

главное значение $ \ log z $ — это значение, полученное из уравнения (\ ref {log2}), когда $ n = 0 $.
и обозначается $ \ text {Log} \, z $.Таким образом
$$ \ text {Log} \, z = \ ln r + i \ Theta. $$
Функция $ \ text {Log} \, z $ корректно определена и однозначна, если $ z \ neq 0 $ и что
$$ \ log z = \ text {Log} \, z + 2n \ pi i \ quad (n \ in \ mathbb Z)
$$
Это сводится к обычному логарифму в исчислении, когда $ z $ — положительное действительное число.

Пример 2: Вычислить $ \ log \ left (1 \ right) $ и $ \ log \ left (-1 \ right) $.

Решение: Из выражения (\ ref {log2})
$$ \ log \ left (1 \ right) = \ ln 1 + i \ left (0 + 2n \ pi \ right) = 2n \ pi i \ quad \ quad (n \ in \ mathbb Z)
$$
а также
$$ \ log \ left (-1 \ right) = \ ln 1 + i \ left (\ pi + 2n \ pi \ right) = \ left (2n + 1 \ right) \ pi i \ quad \ quad (n \ в \ mathbb Z)
$$
Обратите внимание, что $ \ text {Log} \, (1) = 0 $ и $ \ text {Log} \, (-1) = \ pi i $.

Выражение (\ ref {log2}) также эквивалентно следующему:
\ begin {eqnarray *}
\ log z & = & \ ln | z | + i \, \ textbf {arg} (z) \\
& = & \ ln | z | + i \, \ textbf {Arg} (z) + 2ni \, \ pi \ quad \ quad (n \ in \ mathbb Z)
\ end {eqnarray *}

Некоторые основные свойства функции $ \ log z $ следующие:

  1. $ \ log \ left (z_1 \, z_2 \ right) = \ log z_1 + \ log z_2 $
  2. $ \ log \ left (\ dfrac {z_1} {z_2} \ right) = \ log z_1 — \ log z_2 $
  3. Может содержать $ \ text {Log} \, \ left (z_1 \, z_2 \ right) \ neq \ text {Log} \, z_1 + \ text {Log} \, z_2 $

Ветви логарифмов

Из определения (\ ref {log2}) пусть $ \ theta = \ Theta + 2n \ pi $ ($ n \ in \ mathbb Z $), поэтому мы можем написать
\ begin {eqnarray} \ label {log30}
\ журнал г = \ ln г + я \ тета.\ end {eqnarray}

Пусть теперь $ \ alpha $ — любое действительное число. Если мы ограничим значение $ \ theta $ так, чтобы $ \ alpha <\ theta <\ alpha + 2n \ pi $, то функция \ begin {eqnarray} \ label {log3} \ log z = \ ln r + i \ theta \ quad (r> 0, \ alpha <\ theta <\ alpha + 2 \ pi), \ end {eqnarray} с компонентами \ begin {eqnarray *} и (г, \ theta) = \ ln r, \ quad v (r, \ theta) = \ theta, \ end {eqnarray *} - это однозначная и непрерывная функция в указанной области.

Ветвь многозначной функции $ f $ — это любая однозначная функция $ F $, аналитическая в некоторой области в каждой точке $ z $, значение которой $ F (z) $ является одним из значений из $ f $.Требование аналитичности, конечно, не позволяет $ F $ принимать случайный выбор значений $ f $.

Обратите внимание, что для каждого фиксированного α однозначная функция (\ ref {log3}) является ветвью многозначной функции (\ ref {log30}). Функция
\ begin {eqnarray} \ label {log4}
\ text {Log} z = \ ln r + i \ Theta \ quad (r> 0, — \ pi <\ theta <\ pi), \ end {eqnarray} называется главной ветвью.

Разрез ветви — это часть линии или кривой, которая вводится для определения ветви $ F $ многозначной функции $ f $.Точки на разрезе ветвления для $ F $ — это особых точек $ F $, и любая точка, общая для всех разрезов ветвей $ f $, называется точкой ветвления . Начало координат и луч $ \ theta = \ alpha $ составляют сечение ветви (\ ref {log3}) логарифмической функции. Разрез ветви для главной ветви (\ ref {log4}) состоит из начала координат и луча $ = \ pi $. Начало координат, очевидно, является точкой ветвления для ветвей многозначной логарифмической функции.

Мы можем визуализировать многозначную природу $ \ log z $, используя римановы поверхности.Следующие интерактивные изображения показывают действительную и мнимую составляющие $ \ log (z) $. Каждая ветвь мнимой части обозначена разным цветом.

Извините, апплет не поддерживается для маленьких экранов. Поверните устройство в альбомную ориентацию. Или измените размер окна, чтобы оно было больше ширины, чем высоты.

Следует проявлять особую осторожность при использовании ветвей логарифмической функции, особенно потому, что ожидаемые тождества, включающие логарифмы, не всегда переносятся из исчисления.{\ log \ left (4i \ right)} = 4i. $$

ДАЛЕЕ: Римановы поверхности

[вступление, источник, выпуски]

ISBN: 978-0-6485736-0-9
2019

Решение более сложных логарифмических уравнений

Иногда логарифмические уравнения более сложные. Давайте рассмотрим несколько из этих случаев:

1. Если у вас одинаковый логарифм с обеих сторон, их аргументы будут равны
друг другу.

Пример:

Поскольку основание натурального логарифма — е, возведем обе части в степени е.

С обеих сторон e и ln отменяют, оставляя нам следующее:

5x = 4x + 2

Как видите, аргументы (значение в скобках) равны друг другу. Вы можете запомнить этот ярлык или просто следовать обычной процедуре, как мы только что сделали.

Теперь мы можем перейти к решению уравнения:

x = 2

2) Иногда кажущееся решение неверно.

Помните, что для логарифма с положительным основанием аргумент (значение в скобках) должен быть положительным.Таким образом, если решение для x дает отрицательный аргумент, решение недействительно.

Пример:

В этом случае оба аргумента должны быть больше или равны 0. Когда мы найдем наше решение, мы проверим их, чтобы убедиться, что это правда.

Решим уравнение. Поскольку у нас есть общий журнал с обеих сторон, аргументы равны друг другу:

4x = x — 12

3x = -12

x = -4

Давайте вернем -4 в аргументы.Получаем бревно (-16) с обеих сторон. Но это невозможно, так как 10 нельзя возвести в степень и получить отрицательное число. Это неверный ответ.

Таким образом, это уравнение не имеет решения.

3) Иногда необходимо использовать правила журнала.

Пример:

Поскольку журналы добавляются, мы должны умножить их аргументы:

И решить:

Мы должны решить это с помощью формулы корней квадратного уравнения. х $.x = r $ ($ x = \ log r $) и $ y = \ theta + 2k \ pi i $ для некоторого $ k \ in \ mathbb {Z} $. Обратите внимание, что $ log r \ in \ mathbb {R} $ и $ y \ in [y_0, y_0 + 2 \ pi) $ для подходящим образом выбранного $ k $, т.е. $ y = \ arg (z) $, где $ y_0 \ leq \ arg (z) \ leq y_0 + 2 \ pi $. Итак, $ f $ действительно сюръективен.

Из комментариев, сделанных выше, мы можем соответствующим образом определить функцию комплексного натурального логарифма ветви $ A_ {y_0} $ для всех $ z \ in \ mathbb {C} \ setminus \ {0 \} $ по:

(5)

\ begin {align} \ quad \ log (z) = \ log \ mid z \ mid + i \ arg (z) \ end {align}

Где $ y_0 \ leq \ arg (z)

Определение: Комплексный натуральный логарифм в ветви $ A_ {y_0} = \ {z = x + yi: x \ in \ mathbb {R}, y_0 \ leq y

Так как \ (r = | z | \) и \ (\ theta = \ text {arg} (z) \), мы пришли к нашему определению.

Определение

Функция \ (\ text {log} (z) \) определяется как

\ (\ text {log} (z) = \ text {log} (| z |) + i \ text {arg} (z) \),

где \ (\ text {log} (| z |) \) — обычный натуральный логарифм положительного действительного числа.

Примечания.

  1. Поскольку \ (\ text {arg} (z) \) имеет бесконечно много возможных значений, то же самое делает \ (\ text {log} (z) \).
  2. \ (\ text {log} (0) \) не определено. (Оба потому, что \ (\ text {arg} (0) \) не определен, а \ (\ text {log} (| 0 |) \) не определен.)
  3. Выбор ветви для \ (\ text {arg} (z) \) делает \ (\ text {log} (z) \) однозначным. Обычная терминология означает, что мы выбрали ветвь функции журнала.
  4. Основная ветвь журнала происходит из основной ветви arg. То есть

\ (\ text {log} (z) = \ text {log} (| z |) + i \ text {arg} (z) \), где \ (- \ pi <\ text {arg} (z ) \ le \ pi \) (главная ветвь).

Пример \ (\ PageIndex {2} \)

Вычислить все значения \ (\ text {log} (i) \). Укажите, какой из них исходит из основной ветки.

Решение

У нас есть \ (| i | = 1 \) и \ (\ text {arg} (i) = \ dfrac {\ pi} {2} + 2n \ pi \), поэтому

\ (\ text {log} (i) = \ text {log} (1) + i \ dfrac {\ pi} {2} + i 2n \ pi = i \ dfrac {\ pi} {2} + i2n \ pi \), где \ (n \) — любое целое число.

Основная ветвь \ (\ text {arg} (z) \) находится между \ (- \ pi \) и \ (\ pi \), поэтому \ (\ text {Arg} (i) = \ pi / 2 \).Следовательно, значение \ (\ text {log} (i) \) из главной ветви равно \ (i \ pi / 2 \).

Пример \ (\ PageIndex {3} \)

Вычислить все значения \ (\ text {log} (-1 — \ sqrt {3} i) \). Укажите, какой из них исходит из основной ветки.

Решение

Пусть \ (z = -1 — \ sqrt {3} i \). Тогда \ (| z | = 2 \) и в главной ветви \ (\ text {Arg} (z) = -2 \ pi / 3 \). Итак, все значения \ (\ text {log} (z) \) равны

\ (\ text {log} (z) = \ text {log} (2) — i \ dfrac {2 \ pi} {3} + i2n \ pi \).

Значение из основной ветви: \ (\ text {log} (z) = \ text {log} (2) — i 2 \ pi / 3 \).

1.14.1 Рисунки, показывающие \ (w = \ text {log} (z) \) как отображение

На рисунках ниже показаны различные аспекты сопоставления, заданного \ (\ text {log} (z) \).

На первом рисунке мы видим, что точка \ (z \) отображается в (бесконечно) множество значений \ (w \). В этом случае мы показываем \ (\ text {log} (1) \) (синие точки), \ (\ text {log} (4) \) (красные точки), \ (\ text {log} (i) \ ) (синий крест) и \ (\ text {log} (4i) \) (красный крест).Значения в главной ветви находятся внутри заштрихованной области на плоскости \ (w \). Обратите внимание, что значения \ (\ text {log} (z) \) для данного \ (z \) помещаются с интервалами \ (2 \ pi i \) в \ (w \) — плоскости.

Отображение \ (\ text {log} (z): \ text {log} (1), \ text {log} (4), \ text {log} (i), \ text {log} (4i) \ )

На следующем рисунке показано, что главная ветвь журнала отображает проколотую плоскость в горизонтальную полосу \ (- \ pi <\ text {Im} (w) \ le \ pi \). Мы снова показываем значения \ (\ text {log} (1), \ text {log} (4), \ text {log} (i), \ text {log} (4i) \).Поскольку мы выбрали ветку, для каждого журнала отображается только одно значение.

Отображение \ (\ text {log} (z) \): главная ветвь и проколотая плоскость

На третьем рисунке показано, как круги с центром в 0 отображаются на вертикальные линии, а лучи от начала координат отображаются на горизонтальные линии. Если мы ограничимся главной ветвью, то круги будут отображаться в вертикальные отрезки, а лучи — в одну горизонтальную линию в главной (заштрихованной) области плоскости \ (w \).{(\ dfrac {i \ pi} {4})} = \ sqrt {2} \ dfrac {(1 + i)} {\ sqrt {2}} = 1 + i \).

Другое отличное значение — это когда \ (n = 1 \) и дает минус значение чуть выше.

Пример \ (\ PageIndex {5} \)

Кубические корни: вычислить все кубические корни \ (i \). Укажите значение, которое происходит из основной ветви \ (\ text {log} (z) \).

Решение

У нас есть \ (\ text {log} (i) = i \ dfrac {\ pi} {2} + i 2n \ pi \), где \ (n \) — любое целое число. {у}.{y}> 0 $
для любых реальных $ y $,
логарифмическая функция определена только для $ x> 0 $.
В более общем смысле логарифмическая функция — это функция

$$
y = \ mathop {\ rm log} _ {a} x,
$$

где $ a> 0 $ (
$ a \ neq 1 $)
— произвольное основание логарифма; эту функцию можно выразить через $ \ mathop {\ rm ln} x $
по формуле

$$
\ mathop {\ rm log} _ {a} x =
\ frac {\ mathop {\ rm ln} x} {\ mathop {\ rm ln} а}
.
$$

Логарифмическая функция — одна из основных элементарных функций; его график (см. рис.) называется логарифмической кривой.

Рисунок: l060600a

Основные свойства логарифмической функции вытекают из соответствующих свойств экспоненциальной функции и логарифмов; например, логарифмическая функция удовлетворяет функциональному уравнению

$$
\ mathop {\ rm ln} x + \ mathop {\ rm ln} y = \ mathop {\ rm ln} x y.
$$

Логарифмическая функция $ y = \ mathop {\ rm ln} x $
— строго возрастающая функция, а $ \ lim \ limits _ {x \ downarrow 0} \ mathop {\ rm ln} x = — \ infty $,
$ \ lim \ limits _ {x \ rightarrow \ infty} \ mathop {\ rm ln} x = + \ infty $.{2} + a}) + C.
$$

Зависимость между переменными величинами, выраженная логарифмической функцией, впервые была рассмотрена Дж. Нэпьером в 1614 г.

Логарифмическая функция на комплексной плоскости — это бесконечнозначная функция, определенная для всех значений аргумента $ z \ neq 0 $,
и обозначается $ \ mathop {\ rm Ln} z $ (
или $ \ mathop {\ rm ln} z $
если не возникнет путаницы). Однозначная ветвь этой функции, определяемая формулой

$$
\ mathop {\ rm ln} z = \ mathop {\ rm ln} | z | + i \ mathop {\ rm arg} z,
$$

где $ \ mathop {\ rm arg} z $
— главное значение аргумента комплексного числа $ z $,
$ \ pi <\ mathop {\ rm arg} z \ leq \ pi $, называется главным значением логарифмической функции.{1 / n} - 1). $$

Список литературы
[1] S.M. Никольский, «Курс математического анализа», 1 , МИР (1977)
[2] А.И. Маркушевич, «Теория функций комплексного переменного», 1 , Chelsea (1977)

Главное значение логарифма отображает проколотый комплекс $ z $ —
самолет $ (z \ neq 0) $
на полосу $ — \ pi <\ mathop {\ rm Ln} z \ leq \ pi $ в комплексе $ w $ - самолет.Чтобы заполнить $ w $ - плоскости нужно отобразить бесконечно много копий $ z $ - самолет, где для $ n $ - У первой копии $ - \ pi + 2 n \ pi <\ mathop {\ rm arg} z \ leq \ pi + 2 n \ pi $, $ n = 0, \ pm 1, \ точки $. В этом случае $ 0 $ это точка ветвления. Копии составляют так называемую риманову поверхность логарифмической функции. Ясно, что $ \ mathop {\ rm ln} z $ является взаимно однозначным отображением этой поверхности $ (z \ neq 0) $ на $ w $ - самолет. Производная от главного значения равна $ 1 / z $ ( как и в реальном случае) для $ - \ pi <\ mathop {\ rm arg} z <\ pi $.

Вместо $ \ mathop {\ rm ln} $
и $ \ mathop {\ rm Ln} $,
многие западные авторы пост-исчислительной математики используют $ \ mathop {\ rm log} $
и $ \ mathop {\ rm Log} $ (
см. также (редакционные комментарии к) Логарифм числа).

Список литературы
[a1] Дж. Б. Конвей, «Функции одной комплексной переменной», Springer (1973)
[a2] Э. Марсден, «Базовый комплексный анализ», Freeman (1973)
[a3] E.К. Титчмарш, «Теория функций», Oxford Univ. Press (1979)
[a4] S. Saks, A. Zygmund, «Analytic functions», PWN (1952) (Перевод с польского)
[a5] K.R. Стромберг, «Введение в классический реальный анализ», Wadsworth (1981)

Как процитировать эту запись:
Логарифмическая функция. Математическая энциклопедия. URL: http: // encyclopediaofmath.org / index.php? title = Logarithmic_function & oldid = 47701

Эта статья была адаптирована из оригинальной статьи BSE-3 (создателя), которая появилась в Энциклопедии математики — ISBN 1402006098. См. исходную статью

Расширяющие и уплотняющие логарифмы | Колледж алгебры

Результаты обучения

  • Расширьте логарифм, используя комбинацию правил логарифмирования.
  • Сжать логарифмическое выражение до одного логарифма.

Расширяющиеся логарифмы

Взятые вместе, правило произведения, правило частного и правило мощности часто называют «свойствами бревен».«Иногда мы применяем более одного правила, чтобы расширить выражение. Например:

[латекс] \ begin {array} {l} {\ mathrm {log}} _ {b} \ left (\ frac {6x} {y} \ right) \ hfill & = {\ mathrm {log}} _ { b} \ left (6x \ right) — {\ mathrm {log}} _ {b} y \ hfill \\ \ hfill & = {\ mathrm {log}} _ {b} 6 + {\ mathrm {log}} _ {b} x — {\ mathrm {log}} _ {b} y \ hfill \ end {array} [/ latex]

Мы можем использовать правило мощности для расширения логарифмических выражений, включающих отрицательные и дробные показатели. Вот альтернативное доказательство правила частного для логарифмов, использующее тот факт, что обратная величина является отрицательной степенью:

[латекс] \ begin {array} {l} {\ mathrm {log}} _ {b} \ left (\ frac {A} {C} \ right) \ hfill & = {\ mathrm {log}} _ { b} \ left (A {C} ^ {- 1} \ right) \ hfill \\ \ hfill & = {\ mathrm {log}} _ {b} \ left (A \ right) + {\ mathrm {log} } _ {b} \ left ({C} ^ {- 1} \ right) \ hfill \\ \ hfill & = {\ mathrm {log}} _ {b} A + \ left (-1 \ right) {\ mathrm {log}} _ {b} C \ hfill \\ \ hfill & = {\ mathrm {log}} _ {b} A — {\ mathrm {log}} _ {b} C \ hfill \ end {array} [ / латекс]

Мы также можем применить правило произведения, чтобы выразить сумму или разность логарифмов как логарифм произведения. {2}} \ right) [/ latex].{2} -9 \ right)} \ right) [/ латекс].

Показать решение

[латекс] \ frac {1} {2} \ mathrm {ln} \ left (x — 1 \ right) + \ mathrm {ln} \ left (2x + 1 \ right) — \ mathrm {ln} \ left ( x + 3 \ right) — \ mathrm {ln} \ left (x — 3 \ right) [/ латекс]

Уплотняющие логарифмы

Мы можем использовать только что изученные правила логарифмов, чтобы сжимать суммы, разности и произведения с одним и тем же основанием в виде единственного логарифма. Важно помнить, что логарифмы должны иметь одинаковое основание для объединения. Позже мы узнаем, как изменить основание любого логарифма перед сжатием.

Практическое руководство. Для суммы, разности или произведения логарифмов с одинаковым основанием запишите эквивалентное выражение в виде одного логарифма.

  1. Сначала примените свойство power. Определите термины, которые являются произведениями множителей и логарифма, и перепишите каждое как логарифм степени.
  2. Слева направо примените свойства произведения и частного. {4} \ right) [/ latex].

    Попробуй

    Используйте правило мощности для журналов, чтобы записать [latex] 2 {\ mathrm {log}} _ {3} 4 [/ latex] как единый логарифм с ведущим коэффициентом 1.

    Показать решение

    [латекс] {\ mathrm {log}} _ {3} 16 [/ латекс]

    В следующих нескольких примерах мы будем использовать комбинацию правил логарифмирования для сжатия логарифмов.

    Пример: Использование правил произведения и частного для объединения логарифмов

    Запись [латекс] {\ mathrm {log}} _ {3} \ left (5 \ right) + {\ mathrm {log}} _ {3} \ left (8 \ right) — {\ mathrm {log}} _ {3} \ left (2 \ right) [/ latex] как единственный логарифм.

    Показать решение

    Слева направо, поскольку у нас есть добавление двух журналов, мы сначала используем правило продукта:

    [латекс] {\ mathrm {log}} _ {3} \ left (5 \ right) + {\ mathrm {log}} _ {3} \ left (8 \ right) = {\ mathrm {log}} _ {3} \ left (5 \ cdot 8 \ right) = {\ mathrm {log}} _ {3} \ left (40 \ right) [/ latex]

    Это упрощает наше исходное выражение до:

    [латекс] {\ mathrm {log}} _ {3} \ left (40 \ right) — {\ mathrm {log}} _ {3} \ left (2 \ right) [/ latex]

    Используя правило частного:

    [латекс] {\ mathrm {log}} _ {3} \ left (40 \ right) — {\ mathrm {log}} _ {3} \ left (2 \ right) = {\ mathrm {log}} _ {3} \ left (\ frac {40} {2} \ right) = {\ mathrm {log}} _ {3} \ left (20 \ right) [/ latex]

    Попробуй

    Конденсируйте [латекс] \ mathrm {log} 3- \ mathrm {log} 4+ \ mathrm {log} 5- \ mathrm {log} 6 [/ latex].{4} [/ латекс].

    Применение свойств логарифмов

    В химии pH — это показатель кислотности или щелочности жидкости. По сути, это мера концентрации ионов водорода в растворе. Шкала для измерения pH стандартизирована во всем мире, научное сообщество согласовало ее значения и методы их получения.

    Измерения pH могут помочь ученым, фермерам, врачам и инженерам решать проблемы и определять источники проблем.{+} \ right] [/ латекс]. Если концентрация ионов водорода в жидкости увеличится вдвое, как это повлияет на pH?

    Показать решение

    Предположим, что C — исходная концентрация ионов водорода, а P — исходный pH жидкости. Затем [латекс] \ text {P} = — \ mathrm {log} \ left (C \ right) [/ latex]. Если концентрация удвоена, новая концентрация будет 2 C . Тогда pH новой жидкости [латекс] \ text {pH} = — \ mathrm {log} \ left (2C \ right) [/ latex]

    Использование правила произведения журналов

    [латекс] \ text {pH} = — \ mathrm {log} \ left (2C \ right) = — \ left (\ mathrm {log} \ left (2 \ right) + \ mathrm {log} \ left (C \ right) \ right) = — \ mathrm {log} \ left (2 \ right) — \ mathrm {log} \ left (C \ right) [/ latex]

    Поскольку [латекс] P = — \ mathrm {log} \ left (C \ right) [/ latex], новый pH равен

    .

    [латекс] \ text {pH} = P- \ mathrm {log} \ left (2 \ right) \ приблизительно P — 0.301 [/ латекс]

    Когда концентрация ионов водорода увеличивается вдвое, pH снижается примерно на 0,301.

    Попробуй

    Как изменяется pH при уменьшении вдвое концентрации положительных ионов водорода?

    Показать решение

    pH увеличивается примерно на 0,301.

    Внесите свой вклад!

    У вас была идея улучшить этот контент? Нам очень понравится ваш вклад.

    Улучшить эту страницуПодробнее

    3.2. Комплексные числа — Справочник по теоретической физике 0.5 документация

     def arg (x):
    "" "
    Функция аргумента.
    "" "
    из журнала импорта cmath
    вернуть журнал (x) .imag

    def generate_values ​​():
    "" "
    Создайте значения для проверки функции.
    "" "
    из математического импорта sin, cos, pi
    из случайного импорта случайный
    # Создайте 3 круга в комплексной плоскости с диаметрами 0,5, 1 и 2. Мы
    # избегайте специальных значений, таких как -1, +/- i и т. д., потому что они обычно отправляют
    # числовые значения, близкие к сечению ветви, и числовые ошибки, затем
    # переверните знак, e.грамм.:
    # В [1]: sqrt ((- 0.5j) ** 2)
    # Out [1]: -0,5j
    #
    # В [2]: (-0,5j) ** 2
    # Out [2]: (-0.25-0j)
    #
    # В [3]: sqrt (-0,25)
    # Out [3]: 0,5j
    # Здесь [3] - правильное значение, а [1] - неверное, но это происходит из-за
    # к ошибкам округления в [2] (малая отрицательная мнимая часть составляет
    # sqrt () возвращает -0,5j вместо + 0,5j).
    #
    # По этой причине мы выбрали N = 7.
    N = 7
    круг = []
    для n в диапазоне (N): circle.append (cos (2 * pi * n / N) + 1j * sin (2 * pi * n / N))
    values ​​= []
    для n в диапазоне (N): значения.добавить (0,5 * круг [n])
    для n в диапазоне (N): values.append (1.0 * circle [n])
    для n в диапазоне (N): values.append (2,0 * круг [n])
    # Добавьте несколько случайных точек:
    для n в диапазоне (30):
    values.append ((random () - 0,5) * 20 + 1j * (random () - 0,5) * 20)
    возвращаемые значения
    значения = generate_values ​​()

    def feq (a, b, max_relative_error = 1e-12, max_absolute_error = 1e-12):
    "" "
    Возвращает True, если a == b для заданных относительных и абсолютных ошибок, в противном случае
    Ложь.
    "" "
    # если числа достаточно близки (абсолютно), то они равны
    если abs (a-b) abs (a):
    относительная_ошибка = абс (а-б) / абс (б)
    еще:
    относительная_ошибка = абс (а-б) / абс (а)
    #print abs (a-b), relative_error
    вернуть relative_error <= max_relative_error def test_zero1 (слева, справа): "" " Проверяет, что комплексная функция f (x) одной комплексной переменной равна нулю."" " для x в значениях: r = feq (lhs (x), rhs (x)) если не r: print "x lhs (x) rhs (x)" напечатать x, lhs (x), rhs (x) утверждать ложно def test_zero2 (слева, справа): "" " Проверяет, что комплексная функция двух комплексных переменных f (x, y) равна нулю. "" " для x в значениях: для y в значениях: r = feq (lhs (x, y), rhs (x, y)) если не r: print "x y lhs (x, y) rhs (x, y)" напечатать x, y, lhs (x, y), rhs (x, y) утверждать ложно def test_zero3 (слева, справа): "" " Проверяет, что комплексная функция f (x, y, z) трех комплексных переменных равна нулю."" " для x в значениях: для y в значениях: для z в значениях: r = feq (lhs (x, y, z), rhs (x, y, z)) если не r: print "x y z lhs (x, y, z) rhs (x, y, z)" напечатайте x, y, z, lhs (x, y, z), rhs (x, y, z) утверждать ложно def diff (f, z0, theta, eps = 1e-8): h = eps * exp (I * тета) return (f (z0 + h) -f (z0)) / h def diff2 (dfdz, dfdconjz, z0, theta): вернуть dfdz (z0) + dfdconjz (z0) * exp (-2 * I * theta) def test_zero (f, dfdz, dfdconjz, z0, theta, eps = 1e-8): утверждать feq (diff (f, z0, theta, eps), diff2 (dfdz, dfdconjz, z0, theta), max_relative_error = eps * 1e2, max_absolute_error = eps * 1e2) от пола импорта математики, пи из cmath импорт sqrt, exp, log I = 1j углы = [0, пи / 7, пи / 4, пи / 2, 3 * пи / 4, пи] для x в значениях: для теты в углах: test_zero (лямбда x: abs (x), лямбда x: x.сопряженный () / (2 * абс (х)), лямбда x: x / (2 * abs (x)), x, theta) test_zero (лямбда x: журнал (x), лямбда x: 1 / x, лямбда x: 0, x, тета) test_zero (лямбда x: журнал (exp (x-x.

    Добавить комментарий

    Ваш адрес email не будет опубликован. Обязательные поля помечены *

    2024 © Все права защищены.